124
www.MRCPass.com Respiratory خاص شكرفاضل اللي وزميخي خاص شكر الدكتور رياض السيد رياضملف ال ھذا فيفضل ال صاحب الدكتورعمل ال وزميلفاضل ال أخي أشكر أنبد ذلك ك شيخون أشرف الزھري يوسف خالدميتعلي ال سوھاج مستشفى مصر[email protected] https://www.facebook.com/elzohryxp May 2012

Respiratory 2012 Mrcppass

Embed Size (px)

Citation preview

www.MRCPass.com 

 

Respiratory 

 

 شكر خاص

رياض السيد رياضالدكتور  شكر خاص ألخي وزميلي الفاضل

صاحب الفضل في ھذا الملف

أشرف شيخونكذلك البد أن أشكر أخي الفاضل وزميل العمل الدكتور

خالد يوسف الزھري

مصر –مستشفى سوھاج التعليمي

[email protected]

https://www.facebook.com/elzohryxp

May 2012

   

www.MRCPass.com  Respiratory 

Dr. Khalid  Yusuf  Elzohry ‐ Sohag Teaching Hospital ‐ 2012 2

Respiratory Q001  A man presents with symptoms suggestive of a pneumonia. The CXR confirms this.  

 

Which of the following features suggests poor prognosis? 

A. Respiratory rate of 20 

B. Temperature 38°C 

C. Age 60 

D. Urea of 10 

E. MTS score of 9 out of 10 

 

 

 

 

 

 

 

 

 

 

 

 

 

 

 

 

 

 

 

Answer: d) urea of 10. 

 

The CURB‐65 score for poor prognosis in pneumonia are : 

* confusion (defined as an AMT of 8 or less) 

* urea greater than 7 mmol/l 

* respiratory rate of 30 breaths per minute or greater 

* blood pressure less than 90 systolic or diastolic blood pressure 60 or less 

* age 65 or older 

www.MRCPass.com  Respiratory 

Dr. Khalid  Yusuf  Elzohry ‐ Sohag Teaching Hospital ‐ 2012 3

Respiratory Q002  

A 60 year old lady presents with shortness of breath. A chest X ray confirms that she has 

a right sided pleural effusion. An Aspirate was taken and the sample was sent for several 

tests.  

 

Which one of the following is an indication for a chest drain? 

A. High protein 

B. Low glucose 

C. Low LDH 

D. PH <7.2 

E. Blood stains 

 

 

 

 

 

 

 

 

 

 

 

 

 

 

 

 

 

 

 

 

 

Answer: D) pH < 7.2 

 

Although a high protein also points tow ards an exudates, a low pH is the best marker of 

infection in an effusion requiring chest drain insertion. 

www.MRCPass.com  Respiratory 

Dr. Khalid  Yusuf  Elzohry ‐ Sohag Teaching Hospital ‐ 2012 4

Respiratory Q3  

A 75 year old heavy smoker presents to the hospital with breathlessness, wheezing and 

a cough with yellow sputum.  

He has  the  following  investigations: Hb 18 g/dl, WCC 12 x 10^9/l, ABGs show a pH of 

7.38, pO2 of 8.5 kPa, pCO2 of 7 kPa. 

 

What is the diagnosis? 

A. Bronchiectasis 

B. Chronic obstructive pulmonary disease 

C. Mesothelioma 

D. Tuberculosis 

E. Cryptogenic fibrosing alveolitis 

 

 

 

 

 

 

 

 

 

 

 

 

 

 

 

 

 

 

 

 

Answer: b) chronic obstructive pulmonary disease.  

 

This patient has  type  II respiratory  failure, without an acidosis, suggesting chronic CO2 

retention. He also is a heavy smoker and has polycythaemia, making COPD most likely. 

www 

Dr. K

Re 

A 45

yello

infe

coa

 

Whi

 

 

 

 

 

 

 

 

 

 

Ans

 

The

thic

HRC

w.MRCPass.c

Khalid  Yusuf

spiratory

5 year old m

ow sputum

ection. On  e

rse crepitat

ich investig

A. Chrom

B. Bronc

C. Sweat

D. Maxil

E. High r

wer: e) Hig

e  diagnosis 

ckening. 

CT showing 

com 

f  Elzohry ‐ So

y Q4 

man presen

m and occas

examinatio

tions are he

ation will re

mosome test

choscopy 

t test 

llary sinus x 

resolution CT

h resolution

is  bronchie

bronchiect

ohag Teachin

ts with a ch

ional haem

n  he  has  c

eard.  

eveal the di

ting 

rays 

n CT.  

ectasis.  HR

asis 

ng Hospital ‐

hronic coug

optysis. He

clubbing  an

iagnosis? 

CT  is  likely

 

‐ 2012 

h productiv

e also has h

nd  on  auscu

y  to  show 

ve of copiou

ad multiple

ultation  ove

bronchial  d

Res

us amounts 

e episodes o

er  the  lung

dilatation  a

spiratory

of thick 

of chest 

g  bases, 

nd wall 

www 

Dr. K

Re 

A 40

 

Wha

 

 

 

 

 

 

 

 

 

 

 

 

 

 

 

 

Ans

 

The

are 

w.MRCPass.c

Khalid  Yusuf

spiratory

0 year old m

at is his alp

A. 10% 

B. 15% 

C. 25% 

D. 50% 

E. 75% 

wer: D) 50%

e normal ge

MS (75%), 

com 

f  Elzohry ‐ So

y Q5 

man has em

ha 1 antitry

%.  

notype is M

MZ (55%), S

ohag Teachin

mphysema. H

ypsin level li

MM and leve

SS (50%) ZZ

ng Hospital ‐

He is found 

ikely to be?

els of enzym

 (15%.) 

 

‐ 2012 

to have the

me is (100%

e SS phenot

%). The relev

Res

type.  

vant enzym

spiratory

me levels 

www.MRCPass.com  Respiratory 

Dr. Khalid  Yusuf  Elzohry ‐ Sohag Teaching Hospital ‐ 2012 7

Respiratory Q6  

A  70  year  old  man  with  known  chronic  obstructive  pulmonary  disease  (COPD)  is 

admitted with symptoms of worsening breathlessness and confusion. Following a dose 

of  200mg  iv  hydrocortisone  and  two  doses  of  salbutamol  and  atrovent  nebulisers, 

arterial blood gases were taken. 

His arterial blood gases are as follows: pH 7.18, PO2 6.6 kPa , PCO2 12.0 kPa. 

 

What should be done next? 

A. Repeat dose of corticosteroids 

B. Mechanical ventilation 

C. 2 litres oxygen 

D. Monteleukast 

E. Magnesium infusion 2g 

 

 

 

 

 

 

 

 

 

 

 

 

 

 

 

 

 

 

 

Answer: b) Mechanical ventilation.  

 

This patient has type II respiratory failure with acidosis, and hence should be considered 

for ventilation, either straight away or with a trial of NIPPV beforehand. 

www.MRCPass.com  Respiratory 

Dr. Khalid  Yusuf  Elzohry ‐ Sohag Teaching Hospital ‐ 2012 8

Respiratory Q7  

A  40  year  old woman  has  been  breathless  and  is  undergoing  investigations  for  the 

cause. She has a previous history of being a smoker and has a dry cough as well. Her 

ABGs show a pO2 of 9 kPa and pCO2 of 5 kPa. Lung  function  tests show reduced vital 

capacity, reduced TLCO but increased KCO. 

 

What is the likely diagnosis? 

A. COPD 

B. Bronchial obstruction from a tumour 

C. Kyphoscoliosis 

D. Obstructive sleep apnoea 

E. Fibrosing alveolitis 

 

 

 

 

 

 

 

 

 

 

 

 

 

 

 

 

 

Answer: c) kyphoscoliosis.  

 

In  this  scenario,  the  lung  function  indicates  that  the  surface  area  for  exchange  are 

inadequate  (low  TLCO)  hence  suggesting  that  lung  expansion  is  reduced.  How  ever, 

cardiac output is increased to compensate, and henced KCO is increased. Kyphoscoliosis 

would restrict lung expansion, or a neuromuscular disorder (eg spina bifida) could cause 

this. 

www.MRCPass.com  Respiratory 

Dr. Khalid  Yusuf  Elzohry ‐ Sohag Teaching Hospital ‐ 2012 9

Respiratory Q8  

A 50 year old woman is admitted with infective exacerbation of asthma. She responds to 

medical treatment but Aspergillus fumigatus is eventually cultured from her sputum. 

Further investigations show serum total IgE level was elevated at 350 ng/ml (normal 40‐

180 ng/ml), RAST to Aspergillus fumigatus was class III, Aspergillus fumigatus precipitins 

are negative. 

 

What is the most appropriate management? 

A. No change in medication 

B. High dose oral corticosteroids 

C. CT scan of the chest 

D. Chest X ray 

E. Itraconazole 

 

 

 

 

 

 

 

 

 

 

 

 

 

 

Answer: a) No change in medication.  

 

With  negative  Aspergillus  fumigatus  precipitins  and  serum  total  IgE  less  than  1000 

ng/ml, this patient is unlikely to have Allergic Bronchopulmonary Aspergillosis (ABPA). 

Features which are found commonly in asthmatics without ABPA include: 

Positive  immediate skin reactivity to Aspergillus  fumigatus, which  is present  in 20‐30% 

of asthmatics Positive serum precipitins to Aspergillus, which occur in 10% of asthmatics 

without ABPA Recurrent mucoid impaction and atelectasis Peripheral blood eosinophilia 

and elevation of serum total IgE. 

www.MRCPass.com  Respiratory 

Dr. Khalid  Yusuf  Elzohry ‐ Sohag Teaching Hospital ‐ 2012 10

Respiratory Q9  

A 60 year old man presents with small amounts of haemoptysis and breathlessness. He 

does not have pleuritic chest pains and has normal oxygen saturations.  

 

In  considering  the  potential  diagnosis,  haemoptysis  can  occur  with  which  of  the 

following diagnosis? 

A. Pulmonary fibrosis 

B. Melanoma 

C. Goitre 

D. Thymoma 

E. Aspergilloma 

 

 

 

 

 

 

 

 

 

 

 

 

 

 

 

 

 

 

 

 

 

Answer: e) Aspergilloma.  

 

Haemoptysis can be caused by pulmonary embolus, tuberculous infection, aspergilloma, 

bronchial carcinoma, Goodpasture's syndrome and Wegener's granulomatosis. 

www.MRCPass.com  Respiratory 

Dr. Khalid  Yusuf  Elzohry ‐ Sohag Teaching Hospital ‐ 2012 11

Respiratory Q10  

A 30 year old man gives a six‐month history of worsening breathlessness and coughs up 

half a cupful of sputum daily. He was occasionally wheezy with viral illnesses as a young 

child, works in a factory and has smoked 25 cigarettes per day for the last four years.  

 

The most likely diagnosis is: 

A. Lung carcinoma 

B. Bronchiectasis 

C. Asthma Asbestosis 

D. Chronic obstructive pulmonary disease 

E. Rheumatoid lung 

 

 

 

 

 

 

 

 

 

 

 

 

 

 

 

 

 

 

 

 

 

Answer: b) Bronchiectasis.  

 

Bronchiectasis  is most  likely due to the extensive amounts of sputum production.  In a 

young person, cystic fibrosis and hypogammaglobulinaemia should be considered. 

www.MRCPass.com  Respiratory 

Dr. Khalid  Yusuf  Elzohry ‐ Sohag Teaching Hospital ‐ 2012 12

Respiratory Q11  

A  60  year  old man  has  smoked  for  10  years.  He  has  a  longstanding  dry  cough  and 

expiratory wheeze. 

Examination reveals scattered rhonchi and reduced lung expansion.  

 

The most likely diagnosis is: 

A. Asthma 

B. Lung carcinoma 

C. Chronic bronchitis 

D. Bronchiectasis 

E. Extrinsic allergic alveolitis 

 

 

 

 

 

 

 

 

 

 

 

 

 

 

 

 

 

 

 

 

 

 

Answer: c) chronic bronchitis.  

 

The heavy smoking history and wheeze suggests chronic bronchitis. 

www.MRCPass.com  Respiratory 

Dr. Khalid  Yusuf  Elzohry ‐ Sohag Teaching Hospital ‐ 2012 13

Respiratory Q12  

A  14  year  old  boy  develops  red  eye  and  rhinitis  frequently  during  the  start  of  the 

summer.  

 

What is the likely triggering agent? 

A. House dust mite 

B. Grass pollen 

C. Willow pollen 

D. Isocyanates 

E. Coal dust 

 

 

 

 

 

 

 

 

 

 

 

 

 

 

 

 

 

 

 

 

 

Answer: B) grass pollen. 

 

Grass pollen  is released  late May till August, and willow pollen  is released from March 

till June. As the allergic rhinitis is triggered during the start of summer, this is most likely 

to be due to grass pollen. 

www.MRCPass.com  Respiratory 

Dr. Khalid  Yusuf  Elzohry ‐ Sohag Teaching Hospital ‐ 2012 14

Respiratory Q13  

A 65 year old man presents with breathlessness that has got gradually worse over three 

months.  He  has  long‐standing  atrial  fibrillation,  for  which  he  takes  warfarin  and 

amiodarone. 

On examination his pulse  is 100/min  in AF, oxygen saturation was 90% on air. His  JVP 

was not  raised and he has  fine bibasal  crackles. Arterial blood gas  saturation  showed 

mild hypoxia and a pulmonary function test revealed a moderate restrictive picture. 

 

The most likely diagnosis is: 

A. Bronchiectasis 

B. Pulmonary embolism 

C. Pulmonary haemorrhage 

D. Amiodarone induced interstitial lung disease 

E. Congestive cardiac failure 

 

 

 

 

 

 

 

 

 

 

 

 

 

 

 

 

Answer: d) amiodarone induced interstitial lung disease .  

 

The case scenario would fit amiodarone  induced  interstitial  lung disease or pulmonary 

fibrosis.  This may  take  several months  or  years  to  develop.  Lung  function  tests may 

show a restrictive picture with reduced transfer factor. A high resolution CT in this case 

is likely to show diffuse "ground glass" opacities or interlobular septal thickening. 

www.MRCPass.com  Respiratory 

Dr. Khalid  Yusuf  Elzohry ‐ Sohag Teaching Hospital ‐ 2012 15

Respiratory Q14  

A 65 year old man has been confirmed to have lung cancer.  

 

Which form of cancer has the worst prognosis? 

A. Small cell 

B. Squamous cell 

C. Adenocarcinoma 

D. Large cell 

E. Prostate metastasis 

 

 

 

 

 

 

 

 

 

 

 

 

 

 

 

 

 

 

Answer: a) small cell.  

 

Small  cell  lung  cancer  has  the worst  prognosis  out  of  all  lung  cancers  and  is  rarely 

suitable  for surgical resection. Even when diagnosed early  the 2‐year survival  is  in  the 

order of 20‐25%. The main form of therapy is chemotherapy. 

Small  cell  cancer  is  associated with  syndrome  of  inappropriate  antidiuretic  hormone 

(ADH)  and  squamous  cell  cancer  is  associated  with  paraneoplastic  hypercalcaemia 

through  increased parathyroid‐related hormone. 75% of small cell cancers arise  in the 

proximal airways. 

www.MRCPass.com  Respiratory 

Dr. Khalid  Yusuf  Elzohry ‐ Sohag Teaching Hospital ‐ 2012 16

Respiratory Q15  

A  22  year  old  female  presents  with  a  chest  infection.  She  is  unable  to  complete  a 

sentence and her peak flow rate was 40% of her normal  level. She is treated with high 

flow oxygen, nebulised bronchodilators and oral steroids but this is associated with little 

change in her condition.  

 

Which of the following treatments, given intravenously, would be the most appropriate 

for this patient? 

A. Aminophylline 

B. Augmentin 

C. Hydrocortisone 

D. Magnesium 

E. Salbutamol 

 

 

 

 

 

 

 

 

 

 

 

 

 

 

 

 

 

 

Answer: D) Magnesium.  

 

IV magnesium is recommended by the British Thoracic Society in severe exacerbation of 

asthma. A dose of 2g  (8 mmol)  is  given  as  a bolus  iv dose  in  the  acute presentation 

period. 

www.MRCPass.com  Respiratory 

Dr. Khalid  Yusuf  Elzohry ‐ Sohag Teaching Hospital ‐ 2012 17

Respiratory Q16  

A 60 year man has been a smoker of 20 a day for 30 years. He has a cough and difficulty 

in breathing for 3 days which has worsened and he presents to hospital. He has home 

nebulisers but not home oxygen. 

On examination, his oxygen  saturations are 95%, blood pressure 110  / 60 mmHg and 

respiratory rate 32. He has diffuse wheezes and is using his respiratory muscles. Arterial 

blood gases done on 6 litres of oxygen show : 

pH 7.25 

P02 16 kPa 

PCO2 8.2 kPa 

 

What should be the next management step? 

A. Non invasive ventilation 

B. Intravenous antibiotics 

C. Intubation and ventilation 

D. Reduce inspired oxygen concentration 

E. Increase oxygen concentration 

 

 

 

 

 

 

 

 

 

 

Answer: d) reduce inspired oxygen concentration. 

 

This man with Chronic Obstructive Pulmonary Disease (COPD) has blood gases showing 

type II respiratory failure with acidosis. He has been placed on 6 litres of oxygen, which 

is  too much  for  a  patient  with  severe  COPD  as  the  history  of  home  nebuliser  use 

suggests. 

His respiratory drive is suppressed by too much oxygen inspired, and hence reduction to 

a  lower  concentration  (e.g.  1  ‐  2  litres)  to  maintain  a  pO2  above  8.5  kPa  is 

recommended. 

www.MRCPass.com  Respiratory 

Dr. Khalid  Yusuf  Elzohry ‐ Sohag Teaching Hospital ‐ 2012 18

Respiratory Q17  

A 70 year woman has a history of dry cough for 2 months. She has  lost 5 kg of weight 

over the 2 months. Her chest X ray shows a  left apical shadowing. Blood tests reveal a 

raised white cell count of 16. She has not managed to cough up any sputum.  

 

Which test should be performed? 

A. CT scan of the chest 

B. Serum ANCA 

C. Ultrasound of the chest 

D. Kveim test 

E. Bronchoscopy 

 

 

 

 

 

 

 

 

 

 

 

 

 

 

 

 

 

 

 

 

Answer: e) bronchoscopy.  

 

This patient is likely to have TB due to the apical shadowing. In a patient who is unable 

to expectorate sputum, bronchoscopy with  lavage (send for AFB) should be performed 

to confirm the diagnosis. 

www.MRCPass.com  Respiratory 

Dr. Khalid  Yusuf  Elzohry ‐ Sohag Teaching Hospital ‐ 2012 19

Respiratory Q18  

A 65 year old man with emphysema presented with increasing dyspnoea and left‐sided 

pleuritic chest pain. A chest radiograph reveals a  left‐sided pneumothorax with a  lung 

edge measured 5 cm away from the chest wall. 

 

The most appropriate management of his condition is: 

A. Review with daily chest radiographs 

B. Intercostal tube drainage 

C. Refer to a respiratory outpatient clinic 

D. CT chest 

E. Simple aspiration 

 

 

 

 

 

 

 

 

 

 

 

 

 

 

 

 

 

 

 

Answer: b) Intercostal tube drainage.  

 

The  lung  edge measurement  suggests  a  greater  than  50%  pneumothorax.  A  smaller 

pneumothorax may  be  amenable  to  aspiration.  In  the  context  of  chronic  underlying 

respiratory  disease  such  as  chronic  obstructive  pulmonary  disorder  and  a  large 

pneumothorax, patients are best managed by pleural drainage. 

www.MRCPass.com  Respiratory 

Dr. Khalid  Yusuf  Elzohry ‐ Sohag Teaching Hospital ‐ 2012 20

Respiratory Q19  

A 70 year old man attends the hospital with a history of proximal muscle weakness. He 

also gives a history of cough of 12 weeks duration and complains of pain of  the small 

joints of the hands. He has small haemorrhages in the nail folds, but is not clubbed. On 

examination of the chest he has bibasal crackles, and a chest radiograph reveals diffuse 

reticular infiltrates. Lung function tests confirm a restrictive pattern.  

 

What is the likely cause of his interstitial lung disease? 

A. SLE 

B. Dermatomyositis 

C. Ankylosing spondylitis 

D. Cryptogenic fibrosing alveolitis 

E. Amiodarone induced fibrosis 

 

 

 

 

 

 

 

 

 

 

 

 

 

 

 

 

 

Answer: b) Dermatomyositis.  

 

In polymyositis and dermatomyositis patients often develop proximal muscle weakness 

and  of  pain  in  the  small  joints  of  the  fingers.  They  may  have  ragged  cuticles  and 

haemorrhages  at  the  finger  nail  folds.  Interstitial  lung  disease  can  occur. Underlying 

malignancy (lungs, ovaries, breasts and stomach) is present in 5% of cases. 

www 

Dr. K

Re 

A  4

desc

feve

sym

 

Whi

 

 

 

 

 

 

 

 

 

Ans

 

The

or K

dila

Bro

w.MRCPass.c

Khalid  Yusuf

spiratory

40  year  old

cribes  long

er or night s

mptoms. Che

ich is the m

A. High 

B. Exerc

C. Echoc

D. Bronc

E. Serum

wer: a) high

e likely diag

Kartagener'

ted central 

nchiectasis‐

com 

f  Elzohry ‐ So

y Q20 

d  man  is  re

  standing  s

sweats, and

est X ray sho

ost appropr

resolution CT

cise tolerance

cardiography

choscopy an

m precipitins

h resolution

nosis is bro

s syndrome

tubular bro

‐ Airw ay di

ohag Teachin

eferred  for 

sputum pro

d he does no

ows mild bi

riate investi

e test 

d biopsy 

 

n CT.  

onchiectasis

e. HRCT wil

onchi and m

latation on 

ng Hospital ‐

investigati

oduction wh

ot smoke. H

ilateral lowe

igation? 

s, and a fam

ll help  to co

mosaic oliga

 the HRCT

‐ 2012 

ion  of  the 

hich  is  strea

How ever th

er zone sha

mily history 

onfirm  the 

emia were 

21 

cause  of  c

aked with 

here is a fam

dowing.  

is suggestiv

diagnosis, 

seen. 

Res

chronic  cou

blood. The

mily history 

ve of cystic 

if changes 

spiratory

ugh.  He 

re  is no 

of such 

fibrosis 

such as 

www.MRCPass.com  Respiratory 

Dr. Khalid  Yusuf  Elzohry ‐ Sohag Teaching Hospital ‐ 2012 22

Respiratory Q21  

A 40 year old man has a 3 month history of cough and dyspnoea. He smokes 20 a day. 

There is no history of asbestos exposure. 

His WBC count is 24 x 10^9/l with 70% neutrophils and 3.0 x 10^9/l (5%) eosinophils. IgE 

level is elevated. 

He has decreased breath sounds corresponding to parenchymal infiltrates on the CXR. 

 

Which is the best test to confirm the diagnosis? 

A. Sputum for Acid Fast Bacilli 

B. HIV test 

C. Autoimmune screen 

D. Aspergillus RAST 

E. Stool for ova, cysts, parasites 

 

 

 

 

 

 

 

 

 

 

 

 

 

 

 

 

 

Answer: d) Aspergillus RAST.  

 

The  condition  described  is  Allergic  Broncho  Pulmonary  Aspergillosis,  which  is 

commoner  among  asthmatics  and  cystic  fibrosis  patients.  Eosinophilia  and  high  IgE 

levels  are  suggestive  of  this  condition.  RAST  test  for  antibodies  tow  ards  Aspergillus 

confirms the diagnosis. 

www.MRCPass.com  Respiratory 

Dr. Khalid  Yusuf  Elzohry ‐ Sohag Teaching Hospital ‐ 2012 23

Respiratory Q22  

A 45 yearold woman is admitted with a 2‐day history of fever, rigors and breathlessness. 

She looks extremely unwell and is confused, cyanosed, has a respiratory rate of 24/min 

and a systolic blood pressure of 85 mmHg. 

There  is  bronchial  breathing  at  her  right  base,  where  a  chest  radiograph  reveals 

consolidation.  

 

Which would be the most appropriate antibiotic regimen? 

A. Intravenous ceftazidime and intravenous gentamicin 

B. Oral erythromycin 

C. Intravenous cefotaxime and oral erythromycin 

D. Intravenous amoxicillin and oral clarithromycin 

E. Oral amoxicillin and oral clarithromycin 

 

 

 

 

 

 

 

 

 

 

 

 

 

 

 

Answer: c) Intravenous cefotaxime and oral erythromycin.  

 

Severe pneumonia  as defined by  the British  Thoracic  Society  guidelines,  is diagnosed 

when  there  are  two  of  the  following  features:  confusion, urea  >7mmol/l,  respiratory 

rate >30/min, and hypotension (SBP <90mmHg, DBP <60mmHg). 

Appropriate treatment  is with  intravenous antimicrobials: cefuroxime 1.5g three times 

daily or cefotaxime 1g three times daily daily PLUS erythromycin 500 mg four times daily 

or clarithromycin 500mg twice daily. 

www.MRCPass.com  Respiratory 

Dr. Khalid  Yusuf  Elzohry ‐ Sohag Teaching Hospital ‐ 2012 24

Respiratory Q23  

A 72 year old man presents with a history of worsening breathlessness and cough. His 

arterial blood gases show the following recordings when taken at room air: 

pH 7.25 

pO2 6.4kPa 

pCO2 8.9 kPa 

Bicarbonate 31mmol/L 

 

What is the most likely diagnosis? 

A. Acute exacerbation of chronic obstructive pulmonary disease 

B. Obstructive sleep apnoea 

C. Pulmonary oedema 

D. Pulmonary embolus 

E. Pulmonary fibrosis 

 

 

 

 

 

 

 

 

 

 

 

 

 

 

 

 

 

Answer: a) Acute exacerbation of chronic obstructive pulmonary disease.  

 

The blood gas  result would be most compatible with a patient with  severe COPD and 

chronic  type  2  respiratory  failure  with  an  acute  exacerbation  (Hypoxia,  respiratory 

acidodis with raised CO2 and metabolic compensation). 

www.MRCPass.com  Respiratory 

Dr. Khalid  Yusuf  Elzohry ‐ Sohag Teaching Hospital ‐ 2012 25

Respiratory Q24  

A  70  year  old man  presents with  a  chronic  cough. He  is  a  heavy  smoker  of  over  40 

cigarettes a day. CXR shows a peripheral  right‐sided  lesion. A biopsy which was  taken 

shows squamous cell carcinoma. No regional  lymph nodes are  involved. Lung  function 

tests show a FEV1 of less than 1.5 litres.  

 

The recommended treatment is: 

A. Chemotherapy 

B. Radiotherapy 

C. Lobectomy 

D. Pneumectomy 

E. Lung transplant 

 

 

 

 

 

 

 

 

 

 

 

 

 

 

 

 

 

 

 

 

Answer: b) Radiotherapy.  

 

Although surgical treatment  is possible  in non small cell  lung carcinoma, a FEV1 of  less 

than 1.5 litres contraindicates surgery. Hence, radiotherapy is recommended. 

www.MRCPass.com  Respiratory 

Dr. Khalid  Yusuf  Elzohry ‐ Sohag Teaching Hospital ‐ 2012 26

Respiratory Q25  

A 55 year old gentleman has pickw ickian syndrome. He has poor exercise tolerance of 

50 yards and often feels lethargic at work.  

 

Which of the following is the best investigation? 

A. Echocardiography to assess cor pulmonale 

B. CT scan of the chest 

C. Blood gas 

D. Sleep study 

E. Exercise tolerance test 

 

 

 

 

 

 

 

 

 

 

 

 

 

 

 

 

 

 

 

Answer: d) sleep study.  

 

The  diagnosis  of  obstructive  sleep  apnoea  can  be  made  with  a  sleep  study 

(polysomnography).  In  sleep  apnoea,  there  is  gross  obesity  and  airways  obstruction, 

occasionally leading to type II respiratory failure. During the sleep study, > 10 episodes 

of  apneic  episodes  (pauses  in  breathing)  satisfies  the  criteria  for  obstructive  sleep 

apnoea. 

www.MRCPass.com  Respiratory 

Dr. Khalid  Yusuf  Elzohry ‐ Sohag Teaching Hospital ‐ 2012 27

Respiratory Q26  

A 60 year old man smoker of 35 pack years presents with a 6 month history of shortness 

of breath. His past medical history includes diabetes and cervical spondylosis. 

Spirometry shows FEV1 of 1 litre ‐ 65% predicted 

FVC 1.03 litres ‐ 57% predicted 

FEV1/FVC ratio of 95. 

 

How would you interpret the spirometry results? 

A. Normal 

B. Mixed defect 

C. Obstructive defect 

D. Restrictive defect 

E. Suggestive of haemorrhage 

 

 

 

 

 

 

 

 

 

 

 

 

 

 

Answer: d) Restrictive defect.  

 

Reduced FEV1 and FVC with normal FEV1 ratio is compatible with restrictive defect. 

Causes of restrictive lung defect are : 

‐ neurogenic or psychogenic causes 

‐ abnormalities of the thoracic wall 

‐ stiff parenchyma (pulmonary fibrosis) 

‐ loss of lung tissue, e.g. pneumonectomy 

‐ displacement 

www.MRCPass.com  Respiratory 

Dr. Khalid  Yusuf  Elzohry ‐ Sohag Teaching Hospital ‐ 2012 28

Respiratory Q27  

A  65  year  old man  has worsening  breathlessness.  He  has  a  past medical  history  of 

arthritis and palpitations. On admission his oxygen saturations were 90% on air. 

ABGs show pO2 9 kPa. pCO2 3.5 kPa, pH 7.36. 

His chest X ray shows patchy shadowing in both lung peripheries. 

 

Which one of the following drugs is likely to be responsible? 

A. Codeine 

B. Tramadol 

C. Simvastatin 

D. Amiodarone 

E. Prednisolone 

 

 

 

 

 

 

 

 

 

 

 

 

 

 

 

 

 

 

Answer: d) amiodarone.  

 

The  hypoxia  and  X  ray  changes  suggest  pulmonary  fibrosis.  Out  of  the  list  of 

medications, amiodarone is the most likely candidate. A lung function test with transfer 

factor  and  also  high  resolution  CT will  help  to  confirm  the  diagnosis  of  amiodarone 

related pulmonary fibrosis. 

www.MRCPass.com  Respiratory 

Dr. Khalid  Yusuf  Elzohry ‐ Sohag Teaching Hospital ‐ 2012 29

Respiratory Q28  

A  65  year  old man  has  been  diagnosed with  chronic  obstructive  pulmonary  disease 

(COPD). Spirometry confirms severe COPD with a FEV1 of less than 25% predicted. In the 

last year he has been admitted to hospital on 6 occasions with COPD exacerbation.  

 

Which one of the following can help to reduce hospital admissions? 

A. Tiotropium 

B. Monteleukast 

C. Oral theophyllines 

D. Salmeterol 

E. Hydrocortisone 

 

 

 

 

 

 

 

 

 

 

 

 

 

 

 

 

 

 

 

 

Answer: a) Tiotropium. 

 

Severe COPD  is diagnosed  if  the FEV1  is  less or equal  to 30% predicted. Studies have 

shown that patients treated with  long acting anticholinergic (e.g. tiotropium) have few 

er exacerbations per year. 

www.MRCPass.com  Respiratory 

Dr. Khalid  Yusuf  Elzohry ‐ Sohag Teaching Hospital ‐ 2012 30

Respiratory Q29  

An  13  year  old male  has  recent  onset  breathlessness. He  has  a  history  of wheezing 

particularly during the summer when the pollen count is high.  

 

Which of these tests would help to confirm the diagnosis? 

A. Trial of inhaled corticosteroids 

B. Trial of anticholinergics 

C. Serial peak flow measurements 

D. Lung function tests 

E. Chest X ray 

 

 

 

 

 

 

 

 

 

 

 

 

 

 

 

 

 

 

 

Answer: c) serial peak flow measurements.  

 

The history of wheezing during pollen exposure  suggests asthma. The best diagnostic 

test for asthma would be demonstration of variable airways obstruction with serial peak 

flow measurements. Asthmatic patients with exacerbation will demonstrate peak flows 

lower than their predicted peak flow, and also a morning dip. There is also reversibility if 

a bronchodilator (e.g. salbutamol) is administered. 

www.MRCPass.com  Respiratory 

Dr. Khalid  Yusuf  Elzohry ‐ Sohag Teaching Hospital ‐ 2012 31

Respiratory Q30  

A  27  year  old  man  presents  with  sudden  onset  left  sided  pleuritic  chest  pain  and 

breathlessness.  

On examination he is distressed, tachypnoeic and has tracheal displacement to the right 

together with a hyper‐resonant percussion note on the left side. There were no breath 

sounds on the left. Chest x ray shows a < 2 cm left sided pneumothorax.  

 

What is the best management procedure? 

A. High dose oxygen 

B. Intercostal drainage tube 

C. Chemical pleurodesis 

D. Aspiration of pneumothorax 

E. Repeat chest x ray in 6 hours 

 

 

 

 

 

 

 

 

 

 

 

 

 

 

 

 

 

 

Answer: d) aspiration of pneumothorax.  

 

Treatment of a pneumothorax of < 2 cm margin would be initial aspiration. 

This  is  less  painful,  leads  to  a  shorter  duration  of  admission,  reduces  the  need  for 

pleurectomy. 

www.MRCPass.com  Respiratory 

Dr. Khalid  Yusuf  Elzohry ‐ Sohag Teaching Hospital ‐ 2012 32

Respiratory Q31  

A 65 year year old  lady presents with weight  loss and haemoptysis and  is admitted to 

hospital. She has been a heavy smoker for 25 years. A Chest X ray shows a mass in the 

right upper lobe.  

 

Which is the best test to investigate at present? 

A. Sputum cytology 

B. CT scan of the lung 

C. Transbronchial biopsy 

D. Lymph node biopsy 

E. Pleural biopsy 

 

 

 

 

 

 

 

 

 

 

 

 

 

 

 

 

 

 

 

Answer: a) sputum cytology.  

 

The mass seen on the CXR is likely to be a malignancy in view of the history of smoking, 

haemoptysis  and  weight  loss.  The  initial  test  should  be  sputum  cytology,  as  the 

diagnosis  can be  confirmed most easily.  Staging CT  scans  can  then be done  to  guide 

further management after this. 

www 

Dr. K

Re 

A 70

set 

 

Whi

 

 

 

 

 

 

 

 

 

 

 

 

Ans

 

An 

mal

spre

synd

Left

w.MRCPass.c

Khalid  Yusuf

spiratory

0 year old 

of investiga

ich one of th

A. FEV1 

B. Horne

C. Histo

D. Hype

E. Neuro

wer: B) Hor

FEV1 of < 1

ignant  pleu

ead,  vocal 

drome are c

t sided Horn

com 

f  Elzohry ‐ So

y Q32 

man has a 

ations.  

he following

of 1.7 L (50%

er’s syndrom

ry of myocar

rcalcaemia 

opathy affect

rner’s syndr

1.1 L  is a co

ural  effusio

cord  para

contraindic

ner's 

ohag Teachin

diagnosis o

g is a contra

% predicted)

me 

rdial infarctio

ting lower lim

rome. 

ontraindicat

on,  distant 

lysis,  phren

ations to su

ng Hospital ‐

of non smal

aindication 

 

on 

mbs 

tion  for mo

metastases

nic  nerve  p

urgery in lun

‐ 2012 

l cell  lung t

to lung sur

ost cardioth

s,  contralat

paralysis,  H

ng cancer.

 

33 

tumour, an

rgery? 

horacic surg

eral media

Horner's  sy

Res

d has comp

gical proced

stinal  lymp

yndrome,  a

spiratory

pleted a 

dures. A 

ph  node 

nd  SVC 

www.MRCPass.com  Respiratory 

Dr. Khalid  Yusuf  Elzohry ‐ Sohag Teaching Hospital ‐ 2012 34

Respiratory Q33  

A 20 year old male presents with breathlessness and wheezing.  

 

Which of the following is most likely to suggest asthma? 

A. Increased serum IgE 

B. Wheezing induced by smoking 

C. Obstructive picture in the lung function tests 

D. Response to prednisolone 

E. Diurnal PEFR variation > 20% 

 

 

 

 

 

 

 

 

 

 

 

 

 

 

 

 

 

 

 

 

 

 

Answer: e) diurnal PEFR variation of >20%.  

 

In asthma, diurnal PEFR variability is due to various degrees of bronchial hyperreactivity. 

This  is  the  best  indicator  of  likely  asthma.  A  raised  IgE  indicates  atopy  but  is  not 

diagnostic of asthma. 

www.MRCPass.com  Respiratory 

Dr. Khalid  Yusuf  Elzohry ‐ Sohag Teaching Hospital ‐ 2012 35

Respiratory Q34  

A  70  year  old  patient with  COPD  presents with  cough  and  breathlessness which  has 

worsened over 2 months. 

On admission, he has the following arterial blood gas results. 

pH 7.32, pO2 7 kPa, pCO2 8 kPa, HCO3 34 mmol/l, Base Excess ‐1. 

 

Which one of the following biochemical states fits best? 

A. Chronic respiratory acidosis 

B. Chronic respiratory alkalosis 

C. Chronic metabolic acidosis 

D. Chronic metabolic alkalosis 

E. Acute metabolic acidosis 

 

 

 

 

 

 

 

 

 

 

 

 

 

 

 

 

 

 

Answer: a) chronic respiratory acidosis.  

 

There is a mild respiratory acidosis (pH < 3.5 and pCO2 > 6) and base excess is also not 

elevated (in this case only ‐1). There is metabolic compensation, as indicated by a high 

bicarbonate ( >30). The patient's history of COPD suggests that he has chronic hypoxia 

and chronic CO2 retention, hence causing the picture of chronic respiratory acidosis. 

www.MRCPass.com  Respiratory 

Dr. Khalid  Yusuf  Elzohry ‐ Sohag Teaching Hospital ‐ 2012 36

Respiratory Q35  

A breathless 35 year old woman has the following lung function tests: 

FEV1 1.2 L (65%) 

FVC 1.4 L (60%) 

FEV1/FVC ratio = 82%predicted 

TLC = 65% predicted 

RV = 60% predicted 

TLCO = 57% predicted 

KCO = 105% predicted 

 

What is the most likely diagnosis? 

A. COPD 

B. Bronchiectasis 

C. Cystic fibrosis 

D. Scoliosis 

E. Pneumonia 

 

 

 

 

 

 

 

 

 

 

 

 

 

 

Answer: d) Scoliosis.  

 

The  lung  function  tests  show  a  significant  restrictive  defect. Only  kyphoscoliosis  or  a 

pneumonitis may fit this picture but given the normal/high KCO (i.e. after correcting for 

alveolar  volumes),  the most  likely  answer  is  kyphoscoliosis  as  the  gas exchange  after 

correcting for the alveolar volume is high. 

www.MRCPass.com  Respiratory 

Dr. Khalid  Yusuf  Elzohry ‐ Sohag Teaching Hospital ‐ 2012 37

Respiratory Q36  

A 70 year old man presented with shortness of breath. On examination, he had the signs 

of a large right‐sided pleural effusion.  

Investigations revealed: Pleural fluid analysis ‐ protein 65 g/L  

 

What is the most likely cause? 

A. Congestive cardiac failure 

B. Nephrotic syndrome 

C. Non specific pericarditis 

D. Liver cirrhosis 

E. Mesothelioma 

 

 

 

 

 

 

 

 

 

 

 

 

 

 

 

 

 

 

 

 

Answer: e) Mesothelioma.  

 

It  is  a  case of exudative pleural effusion  (protein >30  g/l). Mesothelioma  is  the most 

likely  cause  in  this  case.  Other  causes  are:  malignancies,  infection,  autoimmune 

conditions. 

www.MRCPass.com  Respiratory 

Dr. Khalid  Yusuf  Elzohry ‐ Sohag Teaching Hospital ‐ 2012 38

Respiratory Q37  

A 75 year old man with COPD  is on  long‐term oxygen therapy (LTOT). He complains of 

persistent leg swelling during a routine review . 

His ABG on a supplemental oxygen flow rate of 2 l/min, shows: 

pH of 7.35, pCO2 of 5.7kPa, pO2 of 7.8kPa and HCO3 of 28 mmHg 

 

What should be done? 

A. CXR 

B. Overnight oxygen saturation monitoring 

C. Echocardiogram 

D. Repeat ABG on air 

E. CT scan of the chest 

 

 

 

 

 

 

 

 

 

 

 

 

 

 

 

 

 

 

 

Answer: b) Overnight oxygen saturation monitoring.  

 

The  presence  of  persistent  oedema  or  secondary  polycythaemia  suggests  that  the 

correction of overnight SaO2 may be inadequate. Nocturnal hypoxaemia may be evident 

during an overnight SaO2 monitoring. 

www 

Dr. K

Re 

A  5

exa

of h

A ch

hyp

 

Wha

 

 

 

 

Ans

 

The

crep

retic

che

HRC

opa

w.MRCPass.c

Khalid  Yusuf

spiratory

50  year ma

mination he

his lungs. 

hest X ray r

oxia with a 

at is the bes

A. Lung 

B. Trans

C. Serum

D. High 

E. Serum

wer: d) high

e  diagnosis 

pitations  su

cular opaci

st shows ba

CT of the ch

cities consi

com 

f  Elzohry ‐ So

y Q38 

n  presents

e has clubb

reveals bilat

pO2 of 9 kP

st investiga

function tes

sbronchial bi

m ACE level 

resolution C

m precipitins

h resolution

is  likely  to

uggest  cryp

ties  or  hon

asilar groun

hest showin

stent with ~

ohag Teachin

 with  a  six

ing and the

teral basal s

Pa. 

ation to conf

ts 

opsy 

T of the ches

 

n CT of the c

o  be  pulmo

ptogenic  fib

neycomb  ch

d glass opa

ng basilar g

~Pulmonary

ng Hospital ‐

x month  hi

ere are bilat

shadowing

firm a diagn

st 

chest.  

onary  fibro

brosing  alve

hanges  if  th

cities as we

round glass

y Fibrosis 

‐ 2012 

istory  of  co

teral fine ins

in the lung

nosis? 

osis.  The  si

eolitis.  High

here  is  pulm

ell as linear 

 s opacities 

39 

ough  and  b

spiratory cr

 fields. His 

gns  clubbin

h  resolutio

monary  fib

and reticula

as well as  l

Res

breathlessn

rackles in th

blood gase

ng  and  ins

n  CT  woul

rosis. HRCT

ar opacities

inear and r

spiratory

ess.  On 

he bases 

s reveal 

piratory 

d  show 

T  of  the 

reticular 

www 

Dr. K

Re 

A 40

bee

with

 

Wha

 

 

 

 

Ans

 

Asp

Hae

fung

alle

Asp

w.MRCPass.c

Khalid  Yusuf

spiratory

0 year old m

en  treated  f

hin a cavity 

at is the dia

A. Chlam

B. Capla

C. Extrin

D. Asper

E. Legion

wer: d) Asp

pergillomas 

emoptysis  i

gals or exci

rgic bronch

ergilloma 

com 

f  Elzohry ‐ So

y Q39 

man presen

for  tubercu

in the right

agnosis? 

mydia pneum

an's syndrom

nsic allergic a

rgilloma 

nella infectio

pergilloma. 

are  masse

s  a  commo

sion surger

opulmonar

ohag Teachin

nted with h

losis. Chest

t upper lobe

monia 

me 

alveolitis 

on 

es  of  funga

on  symptom

ry  if  large. T

ry aspergillo

 

ng Hospital ‐

history of re

t x  ray  show

e with an ai

al  mycelia 

m.  They m

They are no

osis). 

‐ 2012 

ecurrent Ha

wed a  roun

r crescent.

that  grow 

may  require

ot associate

40 

aemoptysis.

nded  soft  t

in  preexis

  treatment

ed with bro

Res

 He has pre

issue mass 

sting  lung  c

t  with  eith

onchiectasis

spiratory

eviously 

is  seen 

cavities. 

er  anti‐

s (unlike 

www 

Dr. K

Re 

A  5

hem

unit

Dur

seps

inte

pres

 

Wha

 

 

 

 

 

Ans

 

Res

vasc

con

ARD

w.MRCPass.c

Khalid  Yusuf

spiratory

55  year  old

morrhage an

t. 

ring the foll

sis  persistin

erstitial tissu

ssure was 1

at is the dia

A. Conge

B. Pneum

C. Pulmo

D. Adult

E. Comm

wer: d) adu

piratory  di

cular  perm

dition from

DS 

com 

f  Elzohry ‐ So

y Q40 

d  man  who

nd post‐ope

owing wee

ng  over  sev

ues and he 

13 mmHg. 

agnosis? 

estive cardia

mocystis pne

onary embol

t respiratory 

munity acqui

ult respirato

istress  syn

eability  (ca

m pulmonary

ohag Teachin

o  had  two 

erative seco

ks this was 

veral weeks

became mo

ac failure 

eumonia 

lism 

distress syn

red pneumo

ory distress 

drome  is 

ausing  a  V/

y oedema. I

 

ng Hospital ‐

episodes  o

ondary hem

followed b

s.  Chest  x 

ore significa

drome 

onia 

syndrome. 

associated 

/Q  mismatc

t does not 

‐ 2012 

of  hemorrh

morrhage, w

by bronchop

ray  showed

antly hypox

 

with  profo

ch).  A  norm

respond to 

41 

hagic  shock

was admitte

pneumonia 

d  progressi

ic. Pulmona

ound  hypo

mal  PCWP 

steroids. 

Res

k  due  to  in

d to intens

with symp

ive  changes

ary capillary

xia  and  in

differentia

spiratory

ntestinal 

ive care 

toms of 

s  in  the 

y wedge 

creased 

ates  the 

www.MRCPass.com  Respiratory 

Dr. Khalid  Yusuf  Elzohry ‐ Sohag Teaching Hospital ‐ 2012 42

Respiratory Q41  

A 40 year old man is HIV positive. He presents with breathlessness to the hospital. Chest 

XR  shows  bilateral  interstitial  lung  markings.  His  oxygen  levels  desaturate  upon 

mobilising.  

 

What should be commenced? 

A. Amoxycillin 

B. Amoxycillin and clarithromycin 

C. Iv co‐trimoxazole 

D. Doxycycline 

E. Quadruple TB therapy 

 

 

 

 

 

 

 

 

 

 

 

 

 

 

 

 

 

 

 

 

Answer: c) iv co‐trimoxazole.  

 

The diagnosis  is PCP. Silver staining rather than auramine should be done. Mortality  is 

10%. Iv co‐trimoxazole or pentamidine can be used. Desaturation is typical with exercise 

in PCP. Other bacterial infections such as TB are more common among Africans. 

www 

Dr. K

ReA  2

wor

bud

salb

For 

cou

Exa

Res

Inve

Neu

0.44

The

Wha

Ans

ABP

trea

fung

add

Spu

the 

CXR

w.MRCPass.c

Khalid  Yusuf

spiratory5  year  old 

rsening of a

desonide m

butamol inh

the  past  2

gh and prod

mination a 

piratory exa

estigations 

utrophils  6 

4) x 10^9/l. 

e chest X‐ray

at is the like

A. Chron

B. Allerg

C. Pulmo

D. Extrin

E. Meso

wer: b) Alle

PA  usually  o

ated with  e

gal agents  l

ition skin‐te

tum culture

upper lobe

R showing p

com 

f  Elzohry ‐ So

y Q42 male with

asthma and

etered  dos

halations. 

20  days,  th

duction of m

BP of 130/8

amination r

revealed‐ 

(2‐7.5)  x  1

Serum IgE w

y revealed p

ely diagnosi

nic obstructi

gic broncho p

onary embol

nsic allergic a

thelioma 

ergic bronch

occurs  in  a

either  oral 

like itracon

esting or ex

e is not ofte

es if left unt

ulmonary in

ohag Teachin

h  a  5  year 

 fever of 20

se  inhalatio

he  patient w

mucoid exp

80 mmHg, a

revealed bil

Hemoglobi

0^9/l,  Lym

was 800 (0‐

pulmonary 

is? 

ve pulmonar

pulmonary a

lus 

alveolitis 

ho pulmona

ssociation 

or  inhaled 

azole, but a

xamination 

en helpful. 

reated. 

 nfiltrates on

ng Hospital ‐

long  histor

0 days dura

ons  (400  μg

was  experie

ectoration 

and temper

ateral wide

in‐  12.4  g/

phocytes  3

‐380) kIU/m

infiltrates in

ry disease 

spergillosis

ary aspergill

with  asthm

steroids.  T

amphoteric

of sputum w

ABPA can l

n the right i

‐ 2012 

ry  of  bronc

ation. The p

g/day)  and 

encing  incr

for the past

rature 38.2°

espread whe

/dl,  total  w

3  (1.3‐3.5)  x

ml. 

n the right m

losis. 

ma,  but  can

There  is  som

cin  is not us

with fungal

lead to pro

n ABPA 

43 

chial  asthm

patient was

rarely  eve

reasing  bre

t 20 days. 

°C.  

eezes. 

white  coun

x  10^9/l,  Eo

mid and low

n  occur  in  c

me  evidenc

sed. IgG and

 stains can 

ximal bronc

Res

ma,  presente

well contro

r  required 

athlessness

nt  –  13  x 

osinophils  4

wer zones. 

cystic  fibros

ce  supporti

d  IgE are ra

be used. 

chiectasis a

spiratory

ed with 

olled on 

to  take 

s,  fever, 

10^9/l, 

4  (0.04‐

sis.  It  is 

ng  anti‐

aised. In 

affecting 

www.MRCPass.com  Respiratory 

Dr. Khalid  Yusuf  Elzohry ‐ Sohag Teaching Hospital ‐ 2012 44

Respiratory Q43  

A 16 year girl with know n cystic  fibrosis presents with cough and  fevers. Chest X  ray 

shows right lower zone consolidation.  

 

What antibiotic should be commenced? 

A. Amoxycillin 

B. Metronidazole 

C. Ceftazidime 

D. Gentamicin 

E. Piperacillin 

 

 

 

 

 

 

 

 

 

 

 

 

 

 

 

 

 

 

 

 

 

Answer: C) Ceftazidime 

 

Patients  with  cystic  fibrosis  often  have  pseudomonas  as  a  pathogenic  infection. 

Ceftazidime and ciprofloxacin would cover the organism, and sometimes gentamicin or 

meropenem may be required due to resistance. 

www 

Dr. K

ReA 50

rash

che

Whi

 

Ans

Skin

Chu

affe

The

synd

The

Hyp

and

Pulm

w.MRCPass.c

Khalid  Yusuf

spiratory0 year old a

h. His  routi

st radiograp

ich blood te

A. Total 

B. Antin

C. Anti G

D. Asper

E. Anti‐n

wer: b) Ant

n  rash  and 

urg‐Strauss 

ects small‐ t

e  presence 

drome.  

ese criteria a

(1) asthma 

(2) eosinop

(3) paranas

(4) pulmon

(5) histolog

(6) monone

pergammag

 positive AN

monary Infi

com 

f  Elzohry ‐ So

y Q44 asthmatic tr

ne blood  te

ph is norma

est should b

Ig E level 

eutrophil cy

GBM antibod

rgillus fumig

nuclear antib

tineutrophil

renal  dysf

syndrome,

o medium‐

of  4  or 

are: 

philia of mo

sal sinusitis 

ary infiltrat

gical proof o

euritis mult

lobulinemia

NCA are usu

ltrates in Ch

ohag Teachin

reated with 

ests shows 

al.  

be done nex

toplasmic an

dy 

ates precipit

body (ANA)

l cytoplasm

function  in 

,  or  allergic

sized arteri

more  crite

re than 10%

tes (may be 

of vasculitis 

iplex or pol

a,  increased

ually presen

 hurg Straus

ng Hospital ‐

high dose o

mild  renal 

t? 

ntibodies 

tins 

ic antibodie

an  asthma

c  granulom

es. 

eria  indicat

% in periphe

transient)

with extrav

yneuropath

d  immunog

nt. 

s syndrome

‐ 2012 

of inhaled c

dysfunctio

es.  

atic  suggest

matous  angi

tes  a  high 

eral blood 

vascular eos

hy. 

lobulin E (Ig

45 

corticostero

n and bloo

ts  Churg– 

iitis,  is  a  ra

likelihood 

sinophils 

gE)  levels, r

Res

oids develop

od eosinoph

Strauss  syn

are  syndrom

of  Churg‐

rheumatoid

spiratory

ps a skin 

hilia. His 

ndrome. 

me  that 

‐Strauss 

d factor, 

www.MRCPass.com  Respiratory 

Dr. Khalid  Yusuf  Elzohry ‐ Sohag Teaching Hospital ‐ 2012 46

Respiratory Q45  

A 70 year old man with chronic obstructive  lung disease presents with a cough, fevers 

and green sputum.  

 

What is the antibiotic of choice? 

A. Teichoplanin 

B. Cefotaxime 

C. Erythromycin 

D. Amoxicillin 

E. Ciprofloxacin 

 

 

 

 

 

 

 

 

 

 

 

 

 

 

 

 

 

 

 

 

 

Answer: D) amoxicillin. 

 

In COPD, the 3 bacterial species account for most isolates are : Haemophilus influenzae, 

Streptococcus  pneumoniae  and Moraxella  cattarhalis.  First  line  treatment  should  be 

with amoxicillin, but if the patient is allergic, a tetracycline should be used. 

www 

Dr. K

Re 

A 60

had

synd

 

Whi

 

 

 

 

 

 

 

 

 

Ans

 

Res

mul

narc

failu

ARD

w.MRCPass.c

Khalid  Yusuf

spiratory

0 year old p

 been  in  IT

drome.  

ich of these

A. CT sca

B. Oesop

C. Pulmo

D. Requ

E. Arteri

wer: c) pulm

piratory  di

ltiple blood

cotics. Ches

ure. Hence t

DS 

com 

f  Elzohry ‐ So

y Q46 

patient was 

TU not  long

e would help

an of the che

phageal man

onary capilla

irement for 

ial blood gas

monary cap

istress  synd

 transfusion

st XR would

the best tes

ohag Teachin

involved in

g before  th

p to confirm

est 

nometry 

ary wedge pr

ventilatory s

ses 

pillary wedg

drome  can

ns, drow ni

d show bila

st would be

 

ng Hospital ‐

n a car accid

here was  a 

m the diagno

ressure 

support 

ge pressure.

n  be  caused

ng and aspi

teral  infiltr

e PCWP, wh

‐ 2012 

dent and su

suspicion o

osis? 

  

d  by  sever

iration, and

ates, and th

ich would b

47 

stained mu

of  severe  r

re  trauma, 

d drugs such

he presenta

be normal (<

Res

ultiple fractu

respiratory 

smoke  inh

h as salicyla

ation mimic

<18mmHg)

spiratory

ures. He 

distress 

halation, 

ates and 

cs heart 

www 

Dr. K

Re 

A  6

lesio

 

 

 

 

 

 

 

 

 

 

Ans

 

Squ

bro

Squ

mas

thei

A ca

w.MRCPass.c

Khalid  Yusuf

spiratory

8  year old 

on. Out of t

A. Meso

B. Small

C. Squam

D. Aden

E. Large 

wer: c) squ

uamous  ce

nchogenic c

uamous cell

sses  located

ir vascular s

avitating ca

com 

f  Elzohry ‐ So

y Q47 

man with 

the followin

othelioma 

 cell lung car

mous cell lun

ocarcinoma 

cell carcinom

amous cell 

ll  carcinom

carcinomas

l carcinoma

d  in  the ma

supply and m

rcinomatou

ohag Teachin

weight  loss

ng causes, w

rcinoma 

ng carcinoma

of the lung

ma of the lun

lung carcin

ma  accoun

as  tend  to  f

ain,  lobar o

may have c

us lung lesio

ng Hospital ‐

s  has  a  che

which is mos

ng 

oma. 

ts  for  app

form  firm, 

or segmenta

entral area

 on 

‐ 2012 

est  x  ray w

st likely to c

proximately

nonencaps

al bronchi. 

s of hemorr

48 

which  shows

cause a cavi

y  one‐third

ulated, sha

Larger  tum

rhage, necr

Res

s  a  cavitati

itating lung

d  of  all  ca

arply circum

mors often o

osis or cavit

spiratory

ng  lung 

 lesion? 

ases  of 

mscribed 

outgrow 

tation. 

www.MRCPass.com  Respiratory 

Dr. Khalid  Yusuf  Elzohry ‐ Sohag Teaching Hospital ‐ 2012 49

Respiratory Q48  

A 45 year man has a  long history of productive cough. He had complained of frequent 

chest infections. 

Examination reveals bilateral inspiratory crackles in the bases of the lungs and clubbing 

of the fingers.  

 

Which of following treatments is likely to reduce the frequency of exacerbations? 

A. Prophylactic antibiotics 

B. Inhaled corticosteroids 

C. Oral corticosteroids 

D. Postural drainage 

E. Lung transplant 

 

 

 

 

 

 

 

 

 

 

 

 

 

 

 

 

 

 

 

 

Answer: D) postural drainage.  

 

The  patient  has  bronchiectasis,  in  which  the  common  complication  is  difficulty 

expectorating, postural drainage of secretions is helpful. 

www.MRCPass.com  Respiratory 

Dr. Khalid  Yusuf  Elzohry ‐ Sohag Teaching Hospital ‐ 2012 50

Respiratory Q49  

A 60 year old man, ex‐smoker (20 pack‐years), was admitted to the hospital because of 

a  prolonged  fever  of  up  to  38.5°C  over  a  period  of  10  days,  associated  with  a 

progressively worsening  shortness  of  breath,  a  nonproductive  cough, weakness,  and 

fatigue. 

The  chest  radiograph,  showed  consolidations  at  the  base  of  both  lungs,  prominent 

interstitial markings in the middle lung fields, and hazy infiltrates in the right upper lung 

field. 

A bronchoscopy was done and BAL fluid analysis showed alveolitis with macrophages of 

65%, lymphocytes of 25%, and neutrophils of 10%. 

 

Which drug is most likely to be responsible? 

A. Sulphasalazine 

B. Penicillamine 

C. Gold 

D. Methotrexate 

E. Azathioprine 

 

 

 

 

 

 

 

 

 

 

 

 

 

Answer: d) methotrexate.  

 

The  diagnosis  is  interstitial  pneumonitis. Methotrexate  is  associated  with  interstitial 

pneumonitis. This  is  rare but a serious complication. Diagnosis  is based on  the clinical 

setting,  clinical  manifestations,  radiographic  abnormalities,  bronchoalveolar  lavage 

(BAL), and lung histology. 

www.MRCPass.com  Respiratory 

Dr. Khalid  Yusuf  Elzohry ‐ Sohag Teaching Hospital ‐ 2012 51

Respiratory Q50  

A  40  year  old woman  has  a  history  of  cough with  copious  phlegm with  intermittent 

haemoptysis.  She  also  gets  recurrent  chest  infections  and  has  a  long  history  of  joint 

pains. She is on non‐steroidal anti‐inflammatory drugs. 

Her  chest  radiograph  shows  linear  radioluciences  at  both  bases.  A  high‐resolution 

computed tomography scan (HRCT) of the chest confirms bronchiectasis.  

 

What is the likely cause? 

A. Old tuberculosis 

B. Hypogammaglobulinaemia 

C. Rheumatoid arthritis 

D. Cystic fibrosis 

E. Kartagener's syndrome 

 

 

 

 

 

 

 

 

 

 

 

 

 

 

 

 

 

Answer: c) Rheumatoid arthritis.  

 

The  case  scenario  suggests  rheumatoid  arthritis.  4  %  of  patients  with  rheumatoid 

arthritis  develop  bronchiectasis.  Other  causes  are  chronic  infection  (TB,  measles, 

whooping cough etc),  foreign body aspiration, hypogammaglobulinaemia, Kartagener's 

syndrome, Young's syndrome, cystic fibrosis, allergic bronchopulmonary aspergillosis. 

www.MRCPass.com  Respiratory 

Dr. Khalid  Yusuf  Elzohry ‐ Sohag Teaching Hospital ‐ 2012 52

Respiratory Q51  

A 35 year old lady has a headache, cough and myalgia.  

On examination, she has a pyrexia and scattered crackles bilaterally on auscultation of 

the chest. Rapid cold agglutinins was positive. 

 

What is the likely diagnosis? 

A. Tuberculosis 

B. Staphylococcal pneumonia 

C. Streptococcal pneumonia 

D. Mycoplasma pneumonia 

E. Legionella pneumonia 

 

 

 

 

 

 

 

 

 

 

 

 

 

 

 

 

Answer: d) mycoplasma pneumonia.  

 

Mycoplasma pneumonia is the commonest atypical pneumonia. 

Approximately  15%  of  pneumonias  in  adults  are  due  to  Mycoplasma  pneumoniae. 

Transmission occurs from person to person by infected droplets. The incubation period 

is 9‐21 days. The incidence is higher during the winter months. Fever, chills, cough and 

headache are early symptoms. Dyspnoea, chest pain and haemoptysis are rare. 

Small pleural effusions may occur but are rare. Cold agglutinins are usually present in a 

titre greater than 1:32. 

www.MRCPass.com  Respiratory 

Dr. Khalid  Yusuf  Elzohry ‐ Sohag Teaching Hospital ‐ 2012 53

Respiratory Q52  

A 32 year old man is admitted to hospital with a history of breathlessness of 4‐6 weeks 

duration. He  initially had flu‐like symptoms and was treated by his doctor with a 5‐day 

course of amoxicillin. How ever, he  then started coughing up blood,  leading  to urgent 

referral. 

On examination he was dyspnoeic at rest, with bilateral crackles on auscultation of the 

lungs.  

Investigation revealed anaemia and impaired renal function (creatinine 250 micromol/l). 

Pulmonary  function tests were normal apart  from an abnormally high diffusion  factor. 

Urine dipstick testing showed the presence of red blood cells. 

 

What is the most likely diagnosis? 

A. Goodpasture's syndrome 

B. Farmer's lung 

C. Invasive Aspergillosis 

D. Extrinsic allergic alveolitis 

E. Chronic eosinophilic pneumonia 

 

 

 

 

 

 

 

 

 

 

 

 

Answer: a) Goodpasture's syndrome.  

 

The  clinical  picture  of  pulmonary  and  renal  involvement  is  typical  of  Goodpasture's 

syndrome. The condition is due to the presence of circulating anti‐glomerular basement 

membrane antibodies  (anti‐GBM antibodies. Other causes of pulmonary haemorrhage 

and  renal  failure  include  Wegener's  granulomatosis,  microscopic  polyangiitis  and 

systemic lupus erythematous (SLE). 

www 

Dr. K

Re 

A 40

has 

and

bec

The

 

Wha

 

 

 

Ans

 

ARD

diffe

shad

ven

w.MRCPass.c

Khalid  Yusuf

spiratory

0 year old 

a temperat

  fever desp

ome much 

WCC is 18 x

CRP is 220 

PO2 is 6.5k

Pulmonary 

e chest X‐ray

at is the like

A. Pulm

B. Pneum

C. Sever

D. Pulm

E. Adult 

wer: e) adu

DS can be p

erential  is p

dowing. Pu

tricular fail

com 

f  Elzohry ‐ So

y Q53 

lady presen

ture of 38°C

pite  a  cour

worse over

x 10^9 /l 

mg/l 

kPa despite 

wedge pre

y shows bila

ely diagnosi

onary embo

mothorax 

re pulmonary

onary oedem

respiratory 

ult respirato

precipitated

pulmonary 

ulmonary ca

ure. 

ohag Teachin

nts with bre

C. Over the 

se of oral a

r the last da

high flow O

ssure is 16m

ateral inters

is? 

lus 

y fibrosis 

ma 

distress synd

ory distress 

d by severe

oedema w

apilary wed

 

ng Hospital ‐

eathlessnes

last 5 days 

antibiotics,

ay. 

O2 

mmHg 

stitial lung s

drome 

syndrome. 

e  infection, 

when  the X‐

ge pressure

‐ 2012 

ss, hyperven

she had be

but her pa

shadowing.

 

aspiration 

ray  shows 

e of < 19m

54 

ntilation an

een deterior

artner men

and  illicit d

bilateral  in

mmHg effect

Res

nd tachycard

rating with 

tions  that 

drug use. Th

terstitial  lu

tively exclu

spiratory

dia. She 

a cough 

she has 

he main 

ng  field 

des  left 

www.MRCPass.com  Respiratory 

Dr. Khalid  Yusuf  Elzohry ‐ Sohag Teaching Hospital ‐ 2012 55

Respiratory Q54  

A 30 year old man with bronchial asthma was admitted to hospital for reassessment of 

his recurring dry cough, chest tightness and wheezing. 

Investigations show: 

Neutrophils 6 (2‐7.5) x 10^9/l 

Lymphocytes 3 (1.3‐3.5) x 10^9/l 

Eosinophils 9 (0.04‐0.44) x 10^9/l 

CXR shows reticulonodular shadowing 

Investigations to look for causes of the eosinophilia (e.g., parasitosis, immunodeficiency 

and  malignant  disease)  included  stool  microscopy,  tumour  marker  assays  (for 

carcinoembryonic antigen and cancer antigen (CA) 125, CA 19‐9, CA 15‐3 and CA 72‐4), 

mammography, gastroduodenoscopy, bronchoscopy, abdominopelvic ultrasonography, 

and vaginal and pelvic examination; all yielded normal findings. 

Pulmonary  function tests revealed a mild obstructive and moderate restrictive pattern 

[FEV1] 46%,[FVC] 55%, ratio of FEV1 to FVC 72, peak expiratory flow 53%, [DLCO] 46%. 

 

What is the likely diagnosis? 

A. Tuberculosis 

B. Loeffler's syndrome 

C. Sarcoidosis 

D. Pulmonary embolism 

E. Wegener's granulomatosis 

 

Answer: b) Loeffler's syndrome. 

Hypereosinophilic syndrome is a rare condition where there is an idiopathic eosinophil 

count of > 15 x 10^9/dl. 

It  is associated with Loeffler's syndrome which  is a transient pulmonary reaction with 

nodular or reticular shadowing  (diffuse,  fanshaped shadowing) on chest  radiology and 

eosinophilia. 

Hypereosinophilic  syndrome  generally  affects  young  men  ages  20‐50.  Thrombotic 

tendency,  neurological  involvement  (loss  of  intellect,  depressed  mood  and  poor 

coordination) and restrictive cardiomyopathy occur. There is response to steroids. 

The  lung  involvement  results  in  nocturnal  cough,  productive  sputum,  wheezing  and 

dyspnea, which raises the 

suspicion of bronchial hyperreactivity. Patients may be misdiagnosed as having asthma. 

However, pulmonary function tests typically reveal no airflow limitation. 

www 

Dr. K

Re 

A 40

has 

uns

 

Whi

 

 

Ans

 

Kart

(tra

bro

Sym

rheu

sub

ano

Dex

w.MRCPass.c

Khalid  Yusuf

spiratory

0 year old m

recurrent 

uccessful in

ich one of th

A. Hypo

B. Alpha

C. Defec

D. Karta

E. Asthm

wer: d) Kart

tagener's 

nsposition)

nchiectasis,

mptoms and

umatoid art

clavian  arte

osmia or clu

xtrocardia in

com 

f  Elzohry ‐ So

y Q55 

man has a h

episodes  o

n having chi

he following

gammaglobu

a 1 antitrypsi

ct in CFTR ge

aganer's synd

ma variant 

tagener's sy

syndrome 

  of  the  v

, and immo

d signs are 

thritis, rena

ery.  There 

bbing. 

n Kartagene

ohag Teachin

history of lo

of  chest  inf

ldren. He a

g is likely?

ulinaemia 

in deficiency

ne 

drome 

yndrome. 

is  heredi

viscera,  ab

bility of the

dyspnoea, 

al abnormal

is  also  oti

er's syndrom

ng Hospital ‐

ng standing

fections  and

lso has hea

itary.  It  c

bnormal  fro

e cilia. 

productive

ities, malfo

itis media, 

 me 

‐ 2012 

g cough wit

d  sinusitis. 

ring difficul

comprises 

ontal  sinus

 cough, rec

ormations of

nasal  spee

56 

th sputum p

He  is mar

lty.  

a  triad  o

ses  produc

current  resp

f renal vess

ech,  conduc

Res

production. 

ried  but  ha

of:  situs  i

cing  sinusi

piratory  inf

sels and ano

ctive  heari

spiratory

He also 

as  been 

nversus 

tis  and 

fections, 

omalous 

ng  loss, 

www 

Dr. K

Re 

A 40

that

doe

pur

 

Wha

 

 

 

 

 

 

 

Ans

 

The

diag

out

Seru

mon

Sarc

w.MRCPass.c

Khalid  Yusuf

spiratory

0 year old w

t have not 

es not have 

ple / red no

at is the app

A. Lung 

B. Arran

C. Arran

D. Start 

E. Arran

wer: e) arra

e  combinat

gnostic of  s

patients wi

um  angiote

nitor diseas

coidosis 

com 

f  Elzohry ‐ So

y Q56 

woman is r

responded 

any other s

odules on he

propriate m

Biopsy 

nge CT scan o

nge bronchos

prednisolon

ge outpatien

ange outpat

tion  of  bil

sarcoidosis.

ith  full  lung

ensin‐conve

se. Worseni

ohag Teachin

eferred wit

to a course

symptoms, 

er shins. A c

managemen

of the lungs

scopy and br

nts appointm

tients appo

ateral  hila

. This  is usu

g  function 

erting  enzym

ng disease 

ng Hospital ‐

th a history

e of fluclox

and has ne

chest radiog

nt? 

ronchoalveo

ment for follo

intment for

ar  lymphad

ually  self‐lim

tests  inclu

me  (ACE)  le

can be trea

 

‐ 2012 

y of red, pai

xacillin given

ever smoked

graph show

lar lavage to

ow up 

r follow up.

denopathy 

miting. She

ding  transf

evel  and  lu

ted with pr

57 

inful legs of

n for celluli

d. Examinat

ws prominen

 exclude ma

and  eryth

  should ho

fer  factor  a

ung  function

rednisolone

Res

f 3 weeks d

itis. She  is a

tion reveals

nt hilar regio

lignancy 

hema  nodo

w ever be 

and  lung  v

ns  can  be 

e. 

spiratory

duration 

afebrile, 

s tender 

ons.  

osum  is 

seen  in 

olumes. 

used  to 

www.MRCPass.com  Respiratory 

Dr. Khalid  Yusuf  Elzohry ‐ Sohag Teaching Hospital ‐ 2012 58

Respiratory Q57  

A 50 year old patient has become more breathless over several days. He was assessed 

with a spectrum of tests. The chest X ray was normal. In the interpretation of his arterial 

blood gases on air the following results were obtained:‐ 

pO2: 8.0 kPa 

pCO2: 9.2 kPa 

pH: 7.40 

Base Excess +2 

 

What disease is likely to have caused this? 

A. Community acquired pneumonia 

B. Bronchiectasis 

C. Tuberculosis 

D. Guillain barre syndrome 

E. Small cell carcinoma 

 

 

 

 

 

 

 

 

 

 

 

 

 

 

 

 

 

Answer: d) guillain barre syndrome.  

 

The raised CO2 and hypoxia demonstrate type  II respiratory  failure. The causes of this 

could be obstructive lung disease, neurogenic, or musculoskeletal (kyphoscoliosis). 

www.MRCPass.com  Respiratory 

Dr. Khalid  Yusuf  Elzohry ‐ Sohag Teaching Hospital ‐ 2012 59

Respiratory Q58  

A  40  year  lady  presents  with  a  6  month  history  of  intermittent  haemoptysis.  She 

describes episodes of cutaneous flushing, which typically affects the head and neck. The 

episodes are often associated with an unpleasant warm feeling, itching feeling. They last 

for half an hour. 

She  is  slightly  breathless  but  has  had  no  evidence  of  leg  swelling  or  chest  pain.  She 

smokes 10 cigarettes a day. 

She has  lost 3 kg of weight  in  the past  two months and has been experiencing night 

sweats. A Chest x ray shows sail sign in the left lower lobe, but no visible consolidation.  

 

Which one of the following is the most likely diagnosis? 

A. Wegener's granulomatosis 

B. Goodpasture's syndrome 

C. Carcinoid tumour 

D. Sarcoidosis 

E. Tuberculosis 

 

 

 

 

 

Answer: c) carcinoid tumour. 

 

A sail sign on the CXR  indicates collapse of the  left  lower  lobe. The  likely diagnosis  is a 

carcinoid tumour, which is associated with smoking. 

An early and frequent (94%) symptom of carcinoid tumors is cutaneous flushing, which 

typically affects  the head and neck. Episodes are often associated with an unpleasant 

warm  feeling,  itching,  palpitation,  upper‐body  erythema  and  edema,  salivation, 

diaphoresis,  lacrimation,  and  diarrhea.  Exercise,  stress,  or  certain  foods  (eg,  cheese) 

may trigger an attack. Initial attacks are short, lasting only a few minutes. With time, the 

duration  increases  to  hours.  Flushes  are  reported  to  be  longest  in  association  with 

bronchial carcinoids. 

Carcinoid tumours are neuroendocrine tumours arising from Kultchitzsky cells. They can 

be  central  or  peripheral,  and  are  classified  as  typical  or  atypical  depending  on  their 

histology. They are slow growing tumours of lung with a peak incidence around age 40. 

Bronchial obstruction is common. Diagnosis is by bronchoscopy and biopsy. 

www 

Dr. K

Trea

Sail 

 

w.MRCPass.c

Khalid  Yusuf

atment is us

sign (straig

com 

f  Elzohry ‐ So

sually with 

ght line arou

ohag Teachin

lobectomy.

 und the hea

 

ng Hospital ‐

art border)

‐ 2012 60 

Resspiratory

www.MRCPass.com  Respiratory 

Dr. Khalid  Yusuf  Elzohry ‐ Sohag Teaching Hospital ‐ 2012 61

Respiratory Q59  

A 68  year old man with emphysema  is  referred  to  a  chest  clinic  for  consideration of 

oxygen therapy. Despite maximal treatment with bronchodilators, his exercise tolerance 

is reduced to about 25 yards. 

Physical  examination  and  pulmonary  function  test  was  consistent  with  emphysema. 

There was no evidence of heart failure. ECG was normal. CXR showed hyperinflated lung 

fields. 

Oxygen saturation was 90%. ABG showed ph of 7.36, pCO2 of 3.7kPa, pO2 of 7.6 kPa.  

 

What should be recommended? 

A. Overnight SaO2 monitoring 

B. Course of steroids 

C. Walking test with a trial of oxygen 

D. Repeated ABG in 6 weeks time 

E. Long‐term oxygen therapy 

 

 

 

 

 

 

 

 

 

 

Answer: c) Walking test with a trial of oxygen.  

 

This patient does not meet the criteria for long term oxygen therapy, which is indicated 

when  the paO2  is persistently below 7.3 kPa on air. Clinical  stability  is defined as  the 

absence of exacerbation of chronic lung disease for the previous 5 weeks. 

Patients  without  chronic  hypoxaemia  and  not  on  LTOT,  should  be  considered  for 

ambulatory oxygen therapy if they show evidence of exercise oxygen desaturation (a fall 

of SaO2 of at  least 4% below 90%),  improvement  in exercise capacity with ambulatory 

oxygen therapy and motivation to use the ambulatory oxygen outside the house. 

Assessment should be performed on both air and supplemental oxygen with the patient 

blinded as to the content of the cylinder. 

www.MRCPass.com  Respiratory 

Dr. Khalid  Yusuf  Elzohry ‐ Sohag Teaching Hospital ‐ 2012 62

Respiratory Q60  

A  50  year  old woman  presents with  breathlessness  that  has  been  getting  gradually 

worse over a few weeks and she has difficulty mobilising. On physical examination she is 

found to have a large left sided pleural effusion. 

The presence of the effusion is confirmed by chest radiography.  

 

The most appropriate initial investigation would be: 

A. Diagnostic aspiration of pleural fluid 

B. Sputum cytology 

C. CT chest 

D. Pleural biopsy 

E. Mantoux test 

 

 

 

 

 

 

 

 

 

 

 

 

 

 

Answer: a) Diagnostic aspiration of pleural fluid.  

 

The first investigation should be diagnostic aspiration of pleural fluid. 

Light’s  criteria  can  be  used  to  distinguish  transudates  from  exudates:  in  exudates  at 

least one of the following three criteria are met: 

pleural fluid protein concentration greater than 50% of that in plasma; 

pleural fluid LDH greater than 60% of that in plasma; 

pleural fluid LDH more that two thirds the upper limit of normal in plasma 

In general, if the protein is < 30 g then it is considered to be a transudate, if > 30 g then 

it is an exudate. 

www 

Dr. K

Re 

A 38

is H

doc

 

Wha

 

 

 

Ans

 

PCP

alve

The

3  w

pati

dea

Pne

w.MRCPass.c

Khalid  Yusuf

spiratory

8 year old m

HIV positive

tor makes a

at treatmen

A. AZT 

B. IV me

C. IV co‐

D. IV flu

E. IV am

wer: c) IV c

P patients a

eolar infiltra

e patient sho

weeks  as  w

ients with m

th by over 5

eumocystis p

com 

f  Elzohry ‐ So

y Q61 

man is adm

. His  chest 

a diagnosis 

nt should be

etronidazole 

‐trimoxazole

conazole 

mphotericin 

o‐trimoxazo

re usually h

ates. 

ould receive

well  as  gluc

moderate to

50%. 

pneumonia

ohag Teachin

itted to the

radiograph

of Pneumo

e started?

 

ole. 

hypoxic and

e intraveno

cocorticoids

o severe hy

 

ng Hospital ‐

e hospital w

h  shows bil

cystis carin

d chest radi

us co‐trimo

s  (IV  methy

ypoxia, dec

‐ 2012 

with a one‐w

ateral alveo

ii pneumon

iograph cha

oxazole in a 

ylpred  for 

reases the 

63 

w eek histo

olar  infiltra

nia (PCP).  

aracteristica

dose of 12

3  days).  Th

risk of resp

Res

ry of dyspn

tes. The ad

ally shows b

0mg/kg for 

his,  when 

piratory fail

spiratory

oea. He 

dmitting 

bilateral 

at least 

used  in 

ure and 

www.MRCPass.com  Respiratory 

Dr. Khalid  Yusuf  Elzohry ‐ Sohag Teaching Hospital ‐ 2012 64

Respiratory Q62  

A 40 year old man has a 3 month history of cough, dyspnoea and sputum production. He 

smokes 20 cigarettes a day and has a history of asthma. There is no history of asbestos 

exposure. 

His WBC count is 24 x 10^9/l with 70% neutrophils and 3.0 x 10^9/l (5%) eosinophils. IgE 

level  is  elevated.  He  has  decreased  breath  sounds  corresponding  to  parenchymal 

infiltrates on the CXR. 

 

Which is the best test to confirm the diagnosis? 

A. HIV test 

B. Stool for ova,cysts,parasites 

C. Autoimmune screen 

D. Aspergillus precipitins 

E. Sputum for Acid Fast Bacilli 

 

 

 

 

 

 

 

 

 

 

 

 

Answer: d) Aspergillus precipitins.  

 

The condition described is Allergic Broncho Pulmonary Aspergillosis, commoner among 

asthmatics  and  cystic  fibrosis  patients. Wheeze,  shortness  of  breath  and  productive 

cough are symptoms. 

Allergic bronchopulmonary aspergillosis  results  from an allergic reaction  to Aspergillus 

fumigatus which actually grow s in the walls of the bronchi. 

Eosinophilia and high  IgE  levels are suggestive of  the condition. Aspergillus precipitins 

(lab test to detect antibodies) the diagnosis. The chest radiograph often shows evidence 

of proximal bronchiectasis. 

www.MRCPass.com  Respiratory 

Dr. Khalid  Yusuf  Elzohry ‐ Sohag Teaching Hospital ‐ 2012 65

Respiratory Q63  

A  55  year  old  woman  is  admitted  with  a  history  of  syncopal  episodes  and 

breathlessness. There is no prior history of recent travel. 

On  examination  she  looks  distressed. Her  pulse  rate  is  120  beats  per minute,  blood 

pressure 85/60, JVP is elevated by 5 cm. Respiratory rate 26 per minute, there is a soft 

systolic murmur at the left sternal edge. Breath sounds are clear. 

Investigations show : HB 12.4 g/dL, Blood gases pH 7.42, Pa CO2 3.3 kPa, Pa O2 8.5 kPa. 

She was put on high flow oxygen and given low molecular weight heparin.  

 

The next step in management of this patient should be: 

A. High dose aspirin 

B. Coronary angiography 

C. Urgent CTPA and consider thrombolysis 

D. Intravenous unfractionated heparin 

E. VQ scan 

 

 

 

 

 

 

 

 

 

 

 

 

 

 

 

 

Answer: c) urgent CTPA and consider thrombolysis.  

 

The  diagnosis  massive  central  pulmonary  embolus  as  the  patient  is  hypoxic  and 

hypotensive. The best management is to obtain an urgent CTPA and then thrombolyse if 

there are no contraindications (e.g. high risk of bleeding). 

www 

Dr. K

Re 

A 50

past

mar

 

Whi

 

 

 

 

 

 

Ans

 

The

exp

alte

and

Var

the 

Flow

w.MRCPass.c

Khalid  Yusuf

spiratory

0 year old l

t year. She

rked stridor

ich of the fo

A. FEV1 

B. FVC 

C. Resid

D. Flow 

E. Funct

wer: d) flow

e flow volum

iration  at  d

erations in t

 expiratory 

iable  intrat

expiratory 

w Volume L

com 

f  Elzohry ‐ So

y Q64 

ady has bee

 has previo

r.  

ollowing is h

ual volume 

volume loop

tional residua

w volume lo

me  loop  is a

different  lu

the airway r

limbs of th

horacic  lesi

limb. 

 Loop showin

ohag Teachin

en complai

ously been 

helpful in in

al capacity 

oop.  

a graphic re

ng  volumes

resistance. 

e flow volu

ions are ch

ng airways o

ng Hospital ‐

ning of wor

diagnosed 

vestigating

ecording of

s,  and may

Fixed lesion

me loop. 

aracterized

obstruction

‐ 2012 

rsening diff

with a goit

g the extent 

f airflow du

y  be  affecte

ns cause pla

d by expirat

 

66 

iculty with 

tre. On exa

of airways 

uring maxim

ed  in  a  cha

ateaus in b

tory slowing

Res

breathing o

amination,  s

obstruction

mal respirat

aracteristic 

oth the ins

g and  flatte

spiratory

over the 

she had 

n? 

tion and 

way  by 

piratory 

ening of 

www.MRCPass.com  Respiratory 

Dr. Khalid  Yusuf  Elzohry ‐ Sohag Teaching Hospital ‐ 2012 67

Respiratory Q65  

A  60  year  old  lady  has  arthritic  changes  on  her  hand  joints  of  ulnar  deviation  and 

Butonniere's defomirty. She has chronic breathlessness.  

 

Which of the following is a likely association? 

A. Aspergillosis 

B. Pulmonary fibrosis 

C. Empyema 

D. Lung carcinoma 

E. Pulmonary emboli 

 

 

 

 

 

 

 

 

 

 

 

 

 

 

 

 

 

 

 

Answer: b) pulmonary fibrosis. 

 

The pulmonary complications of rheumatoid arthritis are : 

pulmonary fibrosis (interstitial lung disease) 

bronchiolitis obliterans with organizing pneumonia 

bronchiectasis 

interstitial pneumonitis secondary to drugs 

www.MRCPass.com  Respiratory 

Dr. Khalid  Yusuf  Elzohry ‐ Sohag Teaching Hospital ‐ 2012 68

Respiratory Q66  

A  35  year  old man  has  significant wheezing  and  breathlessness.  Recordings  of  peak 

flows shows diurnal variation. He was prescribed with salbutamol but continues to have 

frequent wheezy episodes.  

 

What is the next step in management? 

A. Phosphodiesterase inhibitors 

B. Leukotriene antagonists 

C. Oral antibiotics 

D. Inhaled corticosteroids 

E. Oral steroids 

 

 

 

 

 

 

 

 

 

 

 

 

 

 

 

 

 

 

 

Answer: d) inhaled corticosteroids. 

 

Diurnal PEFR variation points  tow ards a diagnosis of asthma. First  line  treatment are 

short  acting  B  agonists  such  as  salbutamol  and  Patients  who  do  not  respond  to  B 

agonists  should  be  treated with  inhaled  corticosteroids  (becotide  or  flixotide) which 

help to reduce exacerbations in the long term. 

www.MRCPass.com  Respiratory 

Dr. Khalid  Yusuf  Elzohry ‐ Sohag Teaching Hospital ‐ 2012 69

Respiratory Q67  

A  55  year  old  lady  presents  with  worsening  breathlessness.  She  has  a  history  of 

Raynaud’s phenomenon, heartburn and reflux.  

On  examination,  the  skin  over  her  fingers  is  tight  and  shiny.  She  has  multiple 

telangiectasia  over  her  face  and  nail‐fold  capillary  loops  are  seen. Her  investigations 

show she has Anticentromere antibodies.  

 

What respiratory complication may occur? 

A. Churg Strauss syndrome 

B. Metastatic adenocarcinoma 

C. Pulmonary hypertension 

D. Allergic bronchopulmonary aspergillosis 

E. Mesothelioma 

 

 

 

 

 

 

 

 

 

 

 

 

 

 

 

 

 

Answer: c) pulmonary hypertension.  

 

The patient has  the  clinical  features of  limited  cutaneous  scleroderma  (LcSScformerly 

CREST  syndrome).  Anticentromere  antibodies  occur  in  70‐80%  of  these  patients.  A 

significant  respiratory  complication  is  pulmonary  hypertension.  Treatment  is  with 

prostaglandin analogues e.g. iloprost (intravenous infusions) or bosentan (oral). 

www.MRCPass.com  Respiratory 

Dr. Khalid  Yusuf  Elzohry ‐ Sohag Teaching Hospital ‐ 2012 70

Respiratory Q68  

A 16 year old boy was brought to the A&E with wheeze and extensive rash whilst eating 

at a Chinese takeaway. 

On  examination,  he  had  extensive wheezes  in  his  chest,  stridor,  as well  as  urticaria 

covering his upper and lower limbs. His BP is 82/50 mmHg.  

 

What is the most likely diagnosis? 

A. C1 Esterase Deficiency 

B. Salmonella infection 

C. Idiopathic urticaria 

D. Asthma 

E. Allergy 

 

 

 

 

 

 

 

 

 

 

 

 

 

 

 

 

 

 

 

Answer: E) allergy 

 

The  scenario  is  consistent  with  a  food  allergy,  e.g.  nuts,  leading  to  an  anaphylactic 

reaction.  The  patient  should  be  treated  with  hydrocortisone  (iv  or  im)  as  well  as 

chlorpheniramine. 

www.MRCPass.com  Respiratory 

Dr. Khalid  Yusuf  Elzohry ‐ Sohag Teaching Hospital ‐ 2012 71

Respiratory Q69  

A 40 year old man with pneumonia is being examined.  

 

Which  one  of  the  following  positive  auscultatory  signs  is  diagnostic  of  bronchial 

breathing? 

A. Rhonchi 

B. Increased vocal resonance 

C. Aegophony 

D. Whispering pectoriloquy 

E. Fine inspiratory crepitations 

 

 

 

 

 

 

 

 

 

 

 

 

 

 

 

 

 

 

 

 

 

 

 

Answer: d) Whispering pectoriloquy.  

 

Whispering pectoriloquy is a diagnostic sign for bronchial breathing. 

www.MRCPass.com  Respiratory 

Dr. Khalid  Yusuf  Elzohry ‐ Sohag Teaching Hospital ‐ 2012 72

Respiratory Q70  

A 16 year old boy with previous tuberculosis had a 7‐day history of progressive cough, 

wheeze  and  tachypnoea,  despite  4  days  of  intravenous  flucloxacillin  and  cefotaxime 

therapy. 

Total serum IgE titre was 1600 IU (normal range, 0–180 IU) and the skin prick test was 

positive for Aspergillus fumigatus. 

 

What should he be treated with? 

A. Praziquantel 

B. Aciclovir 

C. Itraconazole 

D. Rifampicin 

E. HAART 

 

 

 

 

 

 

 

 

 

 

 

 

 

 

 

 

Answer: c) itraconazole. 

 

Allergic bronchopulmonary aspergillosis (ABPA) is a diagnosis which can be confirmed by 

significantly elevated serum IgE titre, positive skinprick tests for aspergillus, positive IgG 

aspergillus precipitins. 

Treatment  is with  antifungals.  Steroids may  be  required  in  patients with  respiratory 

distress. 

www 

Dr. K

Re 

A 35

arriv

brea

He 

read

Res

dev

 

Wha

 

 

 

Ans

 

The

into

As t

a la

Larg

w.MRCPass.c

Khalid  Yusuf

spiratory

5 year old m

ved  30 min

athlessness

is cyanosed

ding 81%, d

piratory  ex

viation of th

at immedia

A. Pleur

B. Insert

C. Arran

D. Check

E. Conta

wer: b) Inse

e diagnosis 

o a tension p

the patient 

rge bore ne

ge right side

com 

f  Elzohry ‐ So

y Q71 

man whose 

nutes  earlie

s. He collaps

d and has a

despite high

xamination 

e trachea to

ate course of

odesis 

t large bore n

nge for urgen

k arterial blo

act ITU to arr

ert large bo

is  a  right  s

pneumotho

is unstable,

eedle would

ed pneumot

ohag Teachin

condition h

er  with  a  2

sed while aw

a pulse 130/

 flow oxyge

reveals  re

ow ards the

of action sho

needle into r

nt portable c

ood gases an

range for the

re needle in

sided  spont

orax. 

, there is no

d reduce the

 thorax 

ng Hospital ‐

has suddenl

2‐hour  hist

w aiting CX

/min and B

en via a re‐b

duced  brea

e left.  

ould be take

right hemith

hest XR 

d commence

e patient to b

nto right he

taneous pn

o time to ar

e pressure i

‐ 2012 

y deteriora

tory  of  cen

R. 

BP of 85/45

breathe mas

ath  sounds

en? 

orax 

e BIPAP 

be ventilated

emithorax. 

eumothora

rrange for p

n the right

73 

ted is broug

ntral  pleurit

5 mmHg. Ox

sk. 

  in  the  rig

ax, which h

portable che

hemithorax

Res

ght to A&E.

tic  chest  pa

xygen satur

ght  lung  fie

as now dev

est XR, inse

x. 

spiratory

. He had 

ain  and 

ration  is 

eld  with 

veloped 

ertion of 

www 

Dr. K

Re 

A 4

feve

prev

Ove

sam

His 

 

Wha

 

 

 

 

 

 

 

Ans

 

Prev

wei

exp

Silic

w.MRCPass.c

Khalid  Yusuf

spiratory

5 year old 

ers  and  ha

viously trea

er the past 

mples are AF

chest X‐ray

at is the like

A. Silico

B. React

C. Cadm

D. Histio

E. Asbes

wer: a) silic

vious  TB  p

ght  loss.  C

osure to sil

cosis 

com 

f  Elzohry ‐ So

y Q72 

man devel

s  lost  half

ated tubercu

ten years h

FB negative

y shows nod

ely diagnosi

sis 

tivation of TB

mium lung 

ocytosis X 

stosis 

cosis.  

redisposes 

Coal  miners

ica are at ri

ohag Teachin

ops breath

f  a  stone  i

ulosis. He w

he and his p

dular shadow

is? 

to  silicosis

s,  quarry  w

sk. 

ng Hospital ‐

lessness an

n  weight  o

works as a ta

partner wen

wing in the 

s, which  ca

workers  and

‐ 2012 

nd a non pr

over  the  p

axi driver. 

nt fossil hu

upper zone

n  present 

d  people  w

 

74 

roductive c

ast  six  mo

nting  in old

e. 

as  fever,  b

whose  hobb

Res

ough. He h

onths.  He  h

d quarries. 

breathlessne

bies  predisp

spiratory

has mild 

has  had 

Sputum 

ess  and 

pose  to 

www.MRCPass.com  Respiratory 

Dr. Khalid  Yusuf  Elzohry ‐ Sohag Teaching Hospital ‐ 2012 75

Respiratory Q73  

A  70  year  retired  sandblaster  has  worsening  symptoms  of  cough,  wheeze  and 

breathlessness. He has been keeping parrots for 15 years as a hobby. 

Serum precipitins for M faeni are negative. 

Chest X ray shows hyperinflated lungs. 

His lung function tests show : 

FEV1 1.8L (predicted 2.6) 

FVC 3.0L (predicted 3.2) 

FEV1/FVC 60% 

 

Which of the following is the likely diagnosis? 

A. Silicosis 

B. Chronic obstructive airways disease 

C. Farmer's lung 

D. Allergic bronchopulmonary aspergillosis 

E. Pigeon fancier's lung 

 

 

 

 

 

 

 

 

 

 

 

 

 

 

 

Answer: b) chronic obstructive airways disease.  

 

The  lung  function  tests  show  an  obstructive  picture  (reduced  FEV1/FVC  ratio). 

Interestitial  lung  diseases  are more  likely  to  cause  a  restrictive  picture  on  the  lung 

function test. 

www.MRCPass.com  Respiratory 

Dr. Khalid  Yusuf  Elzohry ‐ Sohag Teaching Hospital ‐ 2012 76

Respiratory Q74  

A 60 year old asthmatic lady is admitted with sudden onset left sided pleuritic chest pain 

accompanied by shortness of breath. 

Arterial blood gases are as follow s: pH of 7.30, p02 of 7.5kPa, and pCO2 of 3.8 kPa. 

Chest XR is normal. She is commenced on oxygen. 

 

What is the most appropriate immediate action? 

A. Request a chest XR in expiration 

B. Request D‐dimer 

C. Start low molecular weight heparin and request CT pulmonary angiography 

D. Start low molecular weight heparin and request a V/Q scan 

E. Broad spectrum antibiotics 

 

 

 

 

 

 

 

 

 

 

 

 

 

 

 

Answer: c) Start low molecular weight heparin and request CT pulmonary angiography. 

 

The symptoms and findings clearly point out tow ards pulmonary embolism (PE). As the 

clinical  probability  of  PE  is  high, measurement  of D‐dimer  should  not  be  performed, 

since the result would not alter the need for definitive investigation. Measurement of D‐

dimer should only be performed when the probability of PE is low , when a normal value 

would be taken as reassuring and further investigation would not be pursued. 

V/Q  scan  less  likely  to  be  unhelpful  in  view  of  her  asthma;  hence  a  CT  pulmonary 

angiogram would be the imaging procedure of choice. 

www.MRCPass.com  Respiratory 

Dr. Khalid  Yusuf  Elzohry ‐ Sohag Teaching Hospital ‐ 2012 77

Respiratory Q75  

A 65 year old man has had 5 kg weight  loss over 6 months. He  is an ex smoker of 25 

cigarettes a day for 40 years and used to work in a coal mine. 

A Chest X ray shows a  large right sided pleural effusion and several pleural plaques  in 

both lung peripheries. 

Pleural aspiration reveals an exudate with 42 g of protein. 

 

What investigation should be recommended? 

A. Bronchoscopy 

B. Lung function tests 

C. Spiral CT of the chest 

D. Thoracoscopy and biopsy 

E. Sputum for AFB 

 

 

 

 

 

 

 

 

 

 

 

 

 

 

Answer: d) thoracoscopy and biopsy.  

 

This patient probably has a malignant effusion as demonstrated by the exudate from the 

pleural effusion. As he  is symptomatic,  the best option would be  to drain  the  fluid as 

well as confirm a diagnosis simultaneously. 

A video assisted thoracoscopy would help to do this. In this procedure, an  illuminated 

tube is inserted through a small incision made betw een the ribs. It allows the operator 

to visualize structures inside the chest and to perform simple procedures such as biopsy 

and nodule excision. 

www.MRCPass.com  Respiratory 

Dr. Khalid  Yusuf  Elzohry ‐ Sohag Teaching Hospital ‐ 2012 78

Respiratory Q76  

A 65 year old man with a smoking history of 50 a day for many years has been a chest 

radiograph showing a lung mass. He is presently waiting a bronchoscopy.  

 

Which one of the following supports a diagnosis of small cell lung cancer? 

A. Disseminated intravascular coagulation 

B. Hypertrophic pulmonary osteoarthropathy (HPOA) 

C. Syndrome of inappropriate antidiuretic hormone secretion (SIADH) 

D. Thrombocytosis 

E. Hypercalcaemia 

 

 

 

 

 

 

 

 

 

 

 

 

 

 

 

 

 

 

 

 

Answer: c) Syndrome of inappropriate antidiuretic hormone secretion (SIADH).  

 

SIADH  is  most  comonly  seen  with  small  cell  carcinoma  rather  than  non‐small  cell 

carcinoma.  HPOA,  hypercalcaemia  without  bone  metastasis  is  more  common  in 

squamous  cell  carcinoma.  DIC  and  thrombocytosis  are  more  common  with 

adenocarcinoma. 

www.MRCPass.com  Respiratory 

Dr. Khalid  Yusuf  Elzohry ‐ Sohag Teaching Hospital ‐ 2012 79

Respiratory Q77  

A  60  year  old  woman  presents  with  a  6‐month  history  of  progressive  shortness  of 

breath. Her past medical history  is unremarkable apart  from Raynaud’s  syndrome  for 

which she takes a calcium channel blocker.  

On examination she has telangiectasia. 

Her chest radiograph shows clear  lung fields, prominent pulmonary arteries and mildly 

enlarged heart. Spirometry is normal, but gas transfer is reduced by 40% of predicted.  

 

What is the most likely diagnosis? 

A. Cor pulmonale 

B. Pulmonary arterial hypertension 

C. Sarcoidosis 

D. Pulmonary emboli 

E. Pulmonary oedema 

 

 

 

 

 

 

 

 

 

 

 

 

 

 

 

 

 

 

Answer: b) Pulmonary arterial hypertension.  

 

The patient is likely to have scleroderma from the clinical history. 

There is also associated pulmonary hypertension which such patients are at risk of. 

www.MRCPass.com  Respiratory 

Dr. Khalid  Yusuf  Elzohry ‐ Sohag Teaching Hospital ‐ 2012 80

Respiratory Q78  

A  75  year  old  lady  is  admitted  with  an  acute  exacerbation  of  chronic  obstructive 

pulmonary  disease  (COPD). One  hour  after  admission  she  remains  distressed with  a 

respiratory rate of 35 per minute and is peripherally cyanosed. Repeated arterial blood 

gases show a severe respiratory acidosis with a pH of <7.2.  

 

Which of the following treatments is recommended? 

A. Give intravenous infusion of aminophylline 

B. Give intravenous hydrocortisone 

C. Repeat bronchodilator therapy and arrange repeat arterial gases 

D. Arrange for noninvasive positive pressure ventilation 

E. High flow oxygen 

 

 

 

 

 

 

 

 

 

 

 

 

 

 

 

 

 

 

Answer: d) Arrange for noninvasive positive pressure ventilation.  

 

Non  invasive  positive  pressure  ventilation  should  be  considered,  especially  in  COPD 

patients  when  there  is  a  need  for  ventilatory  assistance  as  indicated  by  worsening 

dyspnoea, acute respiratory acidosis and worsening oxygenation.  If this does not work 

then intubation and ventilation may be necessary. 

www 

Dr. K

Re 

A 65

of t

 

Whi

airw

 

 

 

 

 

 

 

 

 

 

Ans

 

The

with

Flow

w.MRCPass.c

Khalid  Yusuf

spiratory

5 year old m

he trachea 

ich  of  the  f

ways obstru

A. Flow 

B. Force

C. Force

D. Peak 

E. Resid

wer: A) flow

e flow volum

h a retroste

w volume lo

com 

f  Elzohry ‐ So

y Q79 

man presen

by a retrost

following  in

uction? 

volume loop

ed expiratory

ed vital capac

expiratory fl

ual volume 

w volume lo

me  loop  is t

ernal mass. 

 oop showing

ohag Teachin

nts with ins

ternal goitr

nvestigation

y volume 

city 

low rate 

oop  

the best me

g airways o

ng Hospital ‐

piratory str

e.  

ns  is  the m

ethod of as

bstruction

‐ 2012 

ridor. A che

most  useful 

ssessing ext

81 

est X‐ray sho

to  assess 

tent of obst

Res

owed comp

the  severity

truction ass

spiratory

pression 

y  of  his 

sociated 

www 

Dr. K

Re 

A  2

hist

65%

sim

 

Whi

 

 

 

 

 

 

 

 

 

 

 

Ans

 

α1 

(obs

phe

Smo

w.MRCPass.c

Khalid  Yusuf

spiratory

5  year  old 

ory of  recu

%,  reduced 

ilar present

ich is the lik

A. Eosin

B. Asthm

C. α1 an

D. Fibro

E. Rheum

wer: c) α1 a

antitrypsin

structive  p

enotypes are

oking cessat

com 

f  Elzohry ‐ So

y Q80 

man who 

urrent chest

FEV1, FVC 

tation.  

kely diagnos

ophilic granu

ma 

ntitrypsin def

sing alveolit

matoid lung 

antitrypsin d

n  deficienc

icture)  on 

e M S or Z. 

tion is essen

ohag Teachin

smokes,  h

t  infections

and KCO o

sis? 

uloma 

ficiency 

is 

deficiency.

cy  is  autos

the  lung  f

ZZ has the w

ntial. 

ng Hospital ‐

has  progres

s. His  lung  f

of 45% pred

somal  rece

function  te

worse outco

 

‐ 2012 

sive  breath

function  te

dicted. He 

essive  and 

ests,  with  r

ome. 

82 

hlessness.  H

sts show a 

also has an

causes  an

reduced  tra

Res

He  has  had

FEV1/FVC 

n uncle who

n  emphyse

ansfer  fact

spiratory

d  a  long 

ratio of 

o had a 

ematous 

or.  The 

www.MRCPass.com  Respiratory 

Dr. Khalid  Yusuf  Elzohry ‐ Sohag Teaching Hospital ‐ 2012 83

Respiratory Q81  

A 55 year old woman with  rheumatoid arthritis  is  referred with a history of  recurrent 

chest  infections,  intermittent  wheeze  and  production  of  half  a  teacupful  of  sputum 

daily, on occasions with blood stained.  

 

What is the most likely diagnosis? 

A. Pulmonary fibrosis 

B. Bronchiectasis 

C. Emphysema 

D. Tuberculosis 

E. Squamous lung carcinoma 

 

 

 

 

 

 

 

 

 

 

 

 

 

 

 

 

 

 

 

 

Answer: b) Bronchiectasis.  

 

Bronchiectasis is associated with rheumatoid arthritis, occurring in 3‐4% of patients with 

this  condition.  As with  all  other  causes  of  bronchiectasis,  it  presents with  recurrent 

chest infections and excessive sputum. Recurrent haemoptysis is a common feature. 

www.MRCPass.com  Respiratory 

Dr. Khalid  Yusuf  Elzohry ‐ Sohag Teaching Hospital ‐ 2012 84

Respiratory Q82  

A  38  year  old  man  with  a  history  of  depression  is  brought  to  the  accident  and 

emergency department after being found unconscious in a garage. A friend said that he 

complained of a headache, then had nausea and vomiting. 

He then became unrousable. 

Oxygen saturations are 95% on air and breath sounds are clear on auscultation. 

 

What is the likely diagnosis? 

A. Pulmonary eosinophilia 

B. Pulmonary embolus 

C. Adult respiratory distress syndrome 

D. Pneumocystis pneumonia 

E. Carbon monoxide poisoning 

 

 

 

 

 

 

 

 

 

 

 

 

 

 

Answer: e) Carbon monoxide poisoning. 

 

Carbon  monoxide  poisoning  is  produced  by  the  incomplete  combustion  of  carbon 

containing fuels such as gas, coal, oil, wood and coke. 

Headache is the most common symptom (90%) followed by nausea & vomiting, vertigo, 

alteration in consciousness and weakness. 

The cherry red skin colour occurs when COHb concentration exceeds 20% but it is rarely 

seen  in  life.  Pulse  oximetry  gives  falsely  high  oxygen  saturation  sand  it  is  not 

recommended. 

www.MRCPass.com  Respiratory 

Dr. Khalid  Yusuf  Elzohry ‐ Sohag Teaching Hospital ‐ 2012 85

Respiratory Q83  

A 45 year man has breathlessness and a dry cough. On examination, there are bilateral 

basal crepitations in his lungs. Oxygen saturations are 95% on air. Circulating precipitins 

towards Micropolyspora faeni are positive.  

 

What is the likely diagnosis? 

A. ABPA 

B. Bagassosis 

C. Farmer's lung 

D. Bird fancier's lung 

E. Cheese worker's lung 

 

 

 

 

 

 

 

 

 

 

 

 

 

 

 

 

 

 

 

 

Answer: c) farmer's lung.  

 

This  is  a  form  of  hypersensitivity  pneumonitis.  Farmer's  lung  is  caused  by  the 

actinomycetes Micropolyspora  faeni, and bagassosis  is  caused by Thermoactinomyces 

sacchari. 

www 

Dr. K

Re 

A 4

feve

prev

Ove

sam

His 

 

Wha

 

 

 

 

 

 

 

Ans

 

Prev

wei

exp

Silic

w.MRCPass.c

Khalid  Yusuf

spiratory

5 year old 

ers  and  ha

viously trea

er the past 

mples are AF

chest X‐ray

at is the like

A. Silico

B. Cadm

C. Histio

D. React

E. Asbes

wer: a) silic

vious  TB  p

ght  loss.  C

osure to sil

cosis‐ Mid a

com 

f  Elzohry ‐ So

y Q84 

man devel

s  lost  half

ated tubercu

ten years h

FB negative

y shows nod

ely diagnosi

sis 

mium lung 

ocytosis X 

tivation of TB

stosis 

cosis.  

redisposes 

Coal  miners

ica are at ri

nd Upper Z

ohag Teachin

ops breath

f  a  stone  i

ulosis. He w

he and his p

dular shadow

is? 

to  silicosis

s,  quarry  w

sk. 

Zone linear a

ng Hospital ‐

lessness an

n  weight  o

works as a ta

partner wen

wing in the 

s, which  ca

workers  and

and reticulo

‐ 2012 

nd a non pr

over  the  p

axi driver. 

nt fossil hu

upper zone

n  present 

d  people  w

 onodular sh

86 

roductive c

ast  six  mo

nting  in old

e.  

as  fever,  b

whose  hobb

hadowing, 

Res

ough. He h

onths.  He  h

d quarries. 

breathlessne

bies  predisp

spiratory

has mild 

has  had 

Sputum 

ess  and 

pose  to 

www.MRCPass.com  Respiratory 

Dr. Khalid  Yusuf  Elzohry ‐ Sohag Teaching Hospital ‐ 2012 87

Respiratory Q85  

A  70  year  old man who  has  previously worked  in  the  building  trade presents with  a 

history of  chest pain and dyspnoea. On examination he has evidence of a  right‐sided 

pleural effusion. Pleural aspiration  is performed and a pleural biopsy  taken. Histology 

from the biopsy shows mesothelioma.  

 

What should be done? 

A. The diagnosis should be queried 

B. Radiotherapy should be given to prevent seeding of tumour cells 

C. Surgery 

D. Chemotherapy should be given to prevent seeding of tumour 

E. Curative radiotherapy should be given 

 

 

 

 

 

 

 

 

 

 

 

 

 

 

 

 

 

 

 

Answer: b) Radiotherapy should be given to prevent seeding of tumour cells.  

 

In  mesothelioma  no  treatment  has  been  shown  to  influence  the  universally  fatal 

outcome. After obtaining a positive biopsy, radiotherapy should be given in an attempt 

to prevent seeding of tumour cells, around the area of the biopsy. 

www 

Dr. K

Re 

A 50

to  c

swe

and

CXR

 

The

 

 

 

 

 

 

 

 

 

 

 

Ans

 

TB i

Red

w.MRCPass.c

Khalid  Yusuf

spiratory

0 year old i

casualty. He

eats. On exa

 there are c

R shows fibr

 investigati

A. CT ch

B. Gastr

C. Sputu

D. Mant

E. Fibreo

wer: c) Spu

s likely. In a

d coloured A

com 

f  Elzohry ‐ So

y Q86 

v drug user

e  has  a  chr

amination h

crepitations

rosis and ca

on most like

hest 

ric lavage 

um for acid a

toux test 

optic bronch

tum for aci

a patient wi

AFB 

ohag Teachin

r has been r

ronic  cough

he is unkem

s over the a

vitation in t

ely to confir

and alcohol f

hoscopy 

d and alcoh

th a produc

ng Hospital ‐

referred to 

h  productiv

pt and ema

pex of the l

the left ape

rm the diag

ast bacilli 

hol fast baci

ctive cough,

 

‐ 2012 

the medica

ve  of  sputu

aciated. His 

left lung.  

x. 

gnosis would

illi.  

, AFBs shou

88 

al ward afte

um,  loss  of

trachea is d

d be: 

uld be positi

Res

er being bro

weight,  an

deviated to 

ive in the sp

spiratory

ought in 

nd  night 

the left 

putum. 

www 

Dr. K

Re 

A 5

nod

and

mul

 

Whi

 

 

 

 

 

 

 

 

 

 

 

Ans

 

Mul

can 

sugg

A m

w.MRCPass.c

Khalid  Yusuf

spiratory

5 year old 

dules presen

  transbron

ltinucleated

ich of the fo

A. Histo

B. Asper

C. Tuber

D. Small

E. Silicos

wer: c) tube

ltinucleated

be present

gests either

multinucleat

com 

f  Elzohry ‐ So

y Q87 

smoker has

nt  in  the p

nchial  biop

d giant cells

ollowing is l

plasmosis 

rgillosis 

rculosis 

l cell carcino

sis 

erculosis.  

d giant cells

t in viral inf

r sarcoid, TB

ed giant ce

ohag Teachin

s a history 

erihilar  reg

psy  is  don

.  

likely? 

ma 

s are very la

ections (e.g

B or Wegen

 ll 

ng Hospital ‐

of breathle

gion. His  se

ne.  This  s

arge epider

g. herpes), T

ner's granulo

‐ 2012 

essness and

rum  calcium

shows  non

rmis cells th

TB or lymph

omatosis. 

89 

d a dry cou

m  is norma

n  necrotic 

hat have mu

homa. Gran

Res

gh. He has 

al. A bronch

granulom

ultiple nucle

ulomatous 

spiratory

several 

hoscopy 

as  and 

ei. They 

disease 

www 

Dr. K

Re 

A 3

and

 

Whi

 

 

 

 

 

 

 

 

 

 

 

 

 

Ans

 

Disc

ison

Urin

w.MRCPass.c

Khalid  Yusuf

spiratory

0  year old 

 is about to

ich drug ma

A. Pyraz

B. Eryth

C. Rifam

D. Isonia

E. Etham

wer: c) Rifa

coloration  o

niazid is an e

ne discolour

com 

f  Elzohry ‐ So

y Q88 

woman wi

o be started

ay cause uri

zinamide 

romycin 

mpicin 

azid 

mbutol 

ampicin.  

of  urine  is 

enzyme inh

ration in a p

ohag Teachin

th epilepsy

 on quadru

ine discolou

due  to  rifa

hibitor. 

patient taki

ng Hospital ‐

y has been 

ple therapy

uration? 

ampicin.  Ri

 ng Rifampic

‐ 2012 

diagnosed 

y.  

ifampicin  is

cin 

90 

with pulm

s  a  liver  en

Res

onary  tube

nzyme  indu

spiratory

erculosis 

ucer  but 

www.MRCPass.com  Respiratory 

Dr. Khalid  Yusuf  Elzohry ‐ Sohag Teaching Hospital ‐ 2012 91

Respiratory Q89  

A 30 year old male patient presents with worsening rhinitis, cough and wheezing, which 

has occurred  for  the past 2 years. On admission, his  lung  function  tests show FEV1 of 

55% predicted and FVC of 65% predicted and a chest X ray showed bilateral infiltrates. 

He had an eosinophil count of 5 x 10^9/l (0.04‐0.4) and sputum eosinophilia of 80% was 

also found. There was also elevated IGE levels of >1000 kU/L. Serum ANCA was positive 

at a dilution of 1:1024. 

A nasal biopsy showed chronic  inflammation, with some areas suggestive of vasculitis, 

and eosinophilic infiltration. 

 

Which diagnosis is most likely? 

A. COPD 

B. Mesothelioma 

C. Churg Strauss syndrome 

D. Tuberculosis 

E. Extrinsic allergic alveolitis 

 

 

 

 

 

 

 

Answer: c) Churg Strauss syndrome. 

 

There are many  causes of pulmonary  symptoms with eosinophilia  including  Loeffler's 

syndrome,  Churg  Strauss  syndrome,  pulmonary  eosinophilic  syndrome  and  ABPA. 

Extrinsic  allergic  alveolitis  does  not  cause  a  wheeze  and  also  does  not  cause 

eosinophilia. 

Churg‐Strauss syndrome is an uncommon condition characterised by asthma and blood 

eosinophilia together with an eosinophilic vasculitis. The  initial phase of the disorder is 

one of asthma and allergic rhinitis, often followed by peripheral blood eosinophilia with 

eosinophilic  tissue  disease.  The  vasculitic  phase  that  follows  is  life‐threatening;  how 

ever,  it can often be treated effectively with  immunosuppression.  It  is associated with 

granuloma  formation  and  vasculitis  affecting  several  organs  e.g.  skin,  pericardium, 

kidney and lung. 

www 

Dr. K

Seru

tissu

pati

 

w.MRCPass.c

Khalid  Yusuf

um eosinop

ue biopsy a

ients have c

com 

f  Elzohry ‐ So

philia and e

and  the ant

cANCA and 

ohag Teachin

levated IgE

tineutrophi

about 50% 

 

 

ng Hospital ‐

 levels are 

il  cytoplasm

have pANC

‐ 2012 

typical. Lab

mic antibod

CA. 

92 

boratory dia

y  (ANCA)  t

Res

agnosis is b

est. About 

spiratory

ased on 

25% of 

www 

Dr. K

Re 

A 44

mas

 

Wha

 

 

 

 

 

 

Ans

 

The

may

prec

Asp

w.MRCPass.c

Khalid  Yusuf

spiratory

4 year old m

sses with ai

at is the like

A. tuber

B. Asper

C. Actino

D. Extrin

E. Coal w

wer: B) asp

e  air halo  si

y be predisp

cipitins are 

ergilloma 

com 

f  Elzohry ‐ So

y Q90 

man has pr

r halo arou

ely diagnosi

rculosis 

rgillosis 

omycosis 

nsic allergic a

worker’s lung

ergillosis.  

ign  is partic

posed to by

towards as

ohag Teachin

resented wi

nd them in 

is? 

alveolitis 

cularly asso

y previous T

pergillus. 

 

ng Hospital ‐

ith haemop

the upper z

ociated wit

TB infection 

‐ 2012 

ptysis. He ha

zones. He h

h  the  funga

leading to c

93 

ad a chest 

as positive 

al  infection

cavitation. T

Res

X ray which

serum prec

n  aspergillos

The positive

spiratory

h shows 

cipitins.  

sis, This 

e serum 

www.MRCPass.com  Respiratory 

Dr. Khalid  Yusuf  Elzohry ‐ Sohag Teaching Hospital ‐ 2012 94

Respiratory Q91  

A 55 year lady has had a long history of productive cough and shortness of breath. She 

often coughs up whitish sputum and  is prone  to chest  infections. On examination her 

temperature is 36° C, chest expansion is reduced and there are bilateral wet inspiratory 

crackles.  

 

Which one of the following treatments is most helpful? 

A. Intravenous antibiotics 

B. B agonist inhalers 

C. Postural drainage 

D. Morphine 

E. Prednisolone 

 

 

 

 

 

 

 

 

 

 

 

 

 

 

 

 

 

 

 

 

Answer: c) postural drainage.  

 

This  lady  has  bronchiectasis.  There  is  no  suggestion  of  a  chest  infection  during  this 

episode of admission, hence postural drainage is the best treatment option. 

www.MRCPass.com  Respiratory 

Dr. Khalid  Yusuf  Elzohry ‐ Sohag Teaching Hospital ‐ 2012 95

Respiratory Q92  

A  55  year  old  farmer  has  had  progressive  breathlessness  over  the  past  2  years.  He 

presents with  a  severe  episode  of  dyspnea  and  productive  cough which  occurred  6 

hours after he started working. He mentions that these episodes are typical but today is 

more severe. On admission, his O2 saturations are 94% on air.  

CXR shows bilateral interstitial shadowing in upper zones.  

 

Which of the following is the most useful test? 

A. Precipitins to aspergillus 

B. Sputum for AFB 

C. Kveim test 

D. Precipitins to Micropolyspora faeni 

E. Bronchoscopy 

 

 

 

 

 

 

 

 

 

 

Answer: d) precipitins to Micropolyspora faeni.  

 

This  patient  is  likely  to  have  extrinsic  allergic  alveolitis.  The  classic  presentation  of 

farmer's lung results from inhalational exposure to thermophilic Actinomyces species. 

Patients with extrinsic allergic alveolitis may present acutely with a  flulike  illness with 

cough. They can also present subacutely with recurrent pneumonia or chronically with 

exertional dyspnea, productive cough, and weight loss. 

The onset of symptoms after acute exposure is usually betw een 4 and 12 hours. Some 

antigens  provoke  symptoms  after  repeated  exposure;  these  include  bioaerosols  of 

microbial or animal antigens and a few reactive chemicals. 

Thermophilic  actinomycetes  species which  can  cause  EAA  include  Saccharopolyspora 

rectivirgula  (formerly  Micropolyspora  faeni),  Thermoactinomyces  vulgaris, 

Thermoactinomyces viridis, and Thermoactinomyces sacchari. 

www.MRCPass.com  Respiratory 

Dr. Khalid  Yusuf  Elzohry ‐ Sohag Teaching Hospital ‐ 2012 96

Respiratory Q93  

A 60 year old woman had a CXR showing pulmonary fibrosis. Upon review , of her drug 

history, which of the following drugs might she have been on in the past? 

A. Clarithromycin 

B. Amoxicillin 

C. Busulphan 

D. Omeprazole 

E. Ciprofloxacin 

 

 

 

 

 

 

 

 

 

 

 

 

 

 

 

 

 

 

 

 

 

 

 

 

Answer: C) busulphan.  

 

Busulphan, bleomycin, amiodarone and nitrofurantoin are drugs which commonly cause 

pulmonary fibrosis. 

www 

Dr. K

Re 

A 6

The

afte

An X

in t

faen

 

Wha

 

Ans

Extr

cou

EAA

or c

In 

acti

bird

zon

w h

EAA

w.MRCPass.c

Khalid  Yusuf

spiratory

5 year old 

ese  symptom

er. He is a no

X‐ray of the

he  lower p

ni. 

at is the dia

A. Tube

B. Wege

C. Churg

D. Extrin

E. Pulmo

wer: d) ext

rinsic  allerg

nt is norma

A is a delaye

cell mediate

Extrinsic 

nomycetes 

d fancier's  l

e fibrosis ca

eeze. 

A causing up

com 

f  Elzohry ‐ So

y Q94 

farmer has

ms occur d

on smoker. 

e chest reve

arts of the 

agnosis? 

rculosis 

ener's granul

g Strauss syn

nsic allergic a

onary eosino

rinsic allerg

gic  alveoliti

al. Bronchoa

ed hypersen

ed (type IV) 

Allergic  A

and avian p

ung. Precip

auses audib

pper zone fi

ohag Teachin

s been gett

during work

 

ealed intens

lung. He a

lomatosis 

ndrome 

alveolitis 

ophilia 

gic alveolitis

is  causes  a 

alveolar lava

nsitivity rea

in chronic d

lveolitis,  t

proteins, an

pitins to mic

ble crackles.

ibrosis 

ng Hospital ‐

ing worsen

k  and  are o

sified interst

lso had rais

s.  

neutrophil

age shows l

ction which

disease. 

the  most 

nd the most

cropolyspor

. Symptoms

 

‐ 2012 

ning breathl

often worse

titial lung m

sed serum 

lia due  to  c

lymphocyte

h may be im

common 

t common d

ra faeni are

s are typical

97 

lessness  for

e  for  appro

markings and

precipitins 

cell mediat

es and mast

mmune com

antigens 

diseases are

 seen  in far

lly of breath

Res

r  the past 4

oximately  1

d reticular c

to micropo

ion but  eo

t cells. 

mplex (III) m

are  therm

e farmer's lu

rmer’s  lung

hlessness bu

spiratory

4 years. 

0 hours 

changes 

olyspora 

sinophil 

mediated 

mophilic 

ung and 

g. Upper 

ut not 

www.MRCPass.com  Respiratory 

Dr. Khalid  Yusuf  Elzohry ‐ Sohag Teaching Hospital ‐ 2012 98

Respiratory Q95  

A 22 year old woman has asthma for which she  is using her salbutamol  inhaler two or 

three times a day.  

 

What should be the next step in her management if she worsens? 

A. Advise her to use the salbutamol inhaler regularly three times a day 

B. Add regular inhaled steroid, e.g. beclometasone 100 microg twice daily 

C. Add regular salmeterol twice daily 

D. Add regular inhaled steroid, e.g. beclometasone 1000 microg twice daily 

E. Add regular inhaled steroid, e.g. beclometasone 1000 microg twice daily, plus regular 

long‐acting inhaled beta agonist 

 

 

 

 

 

 

 

 

 

 

 

 

 

 

Answer: b) Add regular inhaled steroid, e.g. beclometasone 100 microg twice daily. 

 

The British Thoracic Society Asthma guidelines are as follows: 

Step 1: PRN use of inhaled short‐acting beta agonists 

Step 2: regular inhaled steroids 

Step 3: high‐dose  inhaled steroids, or  low  ‐dose  inhaled steroids plus  long‐acting beta 

agonist 

Step  4:  high‐dose  inhaled  steroid  and  regular  bronchodilators  (sustained  release 

theophylline,  inhaled  ipatropium,  oral  long‐acting  beta  agonist,  high‐dose 

inhaled bronchodilators, cromoglycate / nedocromil) Step 5: addition of regular 

steroid tablets 

www.MRCPass.com  Respiratory 

Dr. Khalid  Yusuf  Elzohry ‐ Sohag Teaching Hospital ‐ 2012 99

Respiratory Q96  

A 60 year old woman with know n COPD is referred with a 4‐month history of morning 

headaches. She describes a dull headache which  is generalised, worst upon waking up. 

She has home nebuliser (salbutamol and atrovent) but is not on home oxygen. 

A routine arterial blood gas on admission showed a pH of 7.34 pCO2 of 6.2 kPa, pO2 of 

8.8 kPa and HCO3 30 mmHg. 

 

What should be done? 

A. Long‐term oxygen therapy 

B. Ambulatory oxygen therapy 

C. CT chest 

D. Overnight SaO2 and CO2 monitoring 

E. Repeat ABG 

 

 

 

 

 

 

 

 

 

 

 

 

 

Answer: d) Overnight SaO2 and CO2 monitoring.  

 

Morning  headaches  are  often  ascribed  to  patients  with  nocturnal  hypoxia  or  early 

morning  hypercapnia.  They  are  often  associated  with  sleep  apnea  or  a  chronic 

respiratory disease such as COPD. 

The  admission  blood  gases  show mild hypoxia,  and  hypercapnia.  There  is  respiratory 

acidosis  with metabolic  compensation.  It  suggests  that  there  is  a  high  likelihood  of 

chronic hypoxia. 

If  oxygen monitoring  confirmed  nocturnal  hypoxia  (defined  as O2  sats  of  <  90%  for 

more than 30% of sleep time), she may require nocturnal oxygen supplementation. 

www.MRCPass.com  Respiratory 

Dr. Khalid  Yusuf  Elzohry ‐ Sohag Teaching Hospital ‐ 2012 100

Respiratory Q97  

A  couple  attend  the GP  surgery with  their  2  year  old  daughter.  She  has  a  history  of 

repeated  chest  infections,  failure  to  thrive,  and  steatorrhoea  (fatty  stools).  They  also 

have an 8 year old child who is fit and well.  

 

What is the likely diagnosis? 

A. Pulmonary Eosinophilia 

B. Asthma 

C. Cystic fibrosis 

D. Congenital Tuberculosis 

E. Bochdalek Hernia 

 

 

 

 

 

 

 

 

 

 

 

 

 

 

 

 

 

 

 

Answer: c) Cystic fibrosis. 

 

The gene defect in cystic fibrosis is in a mutation on chromosome 7. The inheritance is 

autosomal  recessive  so  the  other  child without  symptoms  has  not  inherited  two  CF 

genes. Pulmonary disease develops over a  few months after birth. Common  infective 

organisms are pneumoccocus, Haemophilus influenzae and Pseudomonas aeruginosa. 

www.MRCPass.com  Respiratory 

Dr. Khalid  Yusuf  Elzohry ‐ Sohag Teaching Hospital ‐ 2012 101

Respiratory Q98  

A 45 year old man complains of wheeziness which is worse. He has a history of asthma. 

His  current medication  consists  of  a  low  dose  of  inhaled  corticosteroids  and  inhaled 

short‐acting beta 2 agonist taken three to four times a day.  

 

What should be done next? 

A. Add oral steroids 

B. Add an inhaled long‐acting beta 2 agonist 

C. Add a long‐acting anticholinergic 

D. Add a short acting anticholinergic 

E. Add a leucotriene receptor antagonist 

 

 

 

 

 

 

 

 

 

 

 

 

 

 

 

 

 

 

Answer: b) Add an inhaled long‐acting beta 2 agonist.  

 

According  to  the British Thoracic Society guidelines  the next  step would be  to add an 

inhaled  long‐acting  beta  2  agonist  (LABA)  and  then  assess  the  situation.  If  there  is  a 

good  response  to  LABA,  this medication  should be continued.  If  there  is benefit  from 

LABA  but  control  is  still  inadequate,  LABA  should  be  continued,  and  the  inhaled 

corticosteroids should be increased to a high dose. 

www.MRCPass.com  Respiratory 

Dr. Khalid  Yusuf  Elzohry ‐ Sohag Teaching Hospital ‐ 2012 102

Respiratory Q99  

A 65 year old heavy smoker is admitted with a history of increasing breathlessness. This 

has been precipitated by the development of a cough with yellow purulent sputum.  

 

The organisms that are likely to cause infective exacerbations of COPD are: 

A. Streptococcus pneumoniae, Legionella pneumophilia, Mycoplasma pneumoniae 

B. Streptococcus pneumoniae, Klebsiella pneumoniae, E coli 

C. Streptococcus pneumoniae, Haemophilus influenzae, Moxarella catarrhalis 

D. Streptococcus pneumoniae, Pseudomonas aeruginosa, Staphylococcus aureus 

E. Streptococcus pneumoniae, Haemophilus influenzae, Legionella pneumophilia 

 

 

 

 

 

 

 

 

 

 

 

 

 

 

 

 

 

 

 

 

 

 

Answer: c) Streptococcus pneumoniae, Haemophilus influenzae, Moxarella catarrhalis. 

 

In  infective  exacerbations  of  COPD,  Streptococcus  pneumoniae  and  Haemophilus 

influenzae, as well as Moraxella are the commonest organisms. 

www.MRCPass.com  Respiratory 

Dr. Khalid  Yusuf  Elzohry ‐ Sohag Teaching Hospital ‐ 2012 103

Respiratory Q100  

A 65 year old man has longstanding breathlessness on exertion. He has been smoking 20 

cigarettes a day over a period of 30 years.  

On  examination,  the  patient  is  breathless with  use  of  accessory muscles  and  resting 

activation of the abdominal muscles. The chest is barrel shaped.  

 

With  regards  to  this  case, which  one  of  the  following  is  know  n  to  be  a  predictor  of 

mortality? 

A. Spirometry 

B. Fibrotic changes 

C. Body Mass Index 

D. Arterial blood gases 

E. How many cigarettes were smoked 

 

 

 

 

 

 

 

 

 

 

 

 

 

 

 

 

 

 

 

Answer: c) Body Mass Index.  

 

A  low  body  mass  index  is  a  known  predictor  of  mortality  in  patients  with  chronic 

obstructive pulmonary disease. 

www.MRCPass.com  Respiratory 

Dr. Khalid  Yusuf  Elzohry ‐ Sohag Teaching Hospital ‐ 2012 104

Respiratory Q101  

A 40 year man presents with shortness of breath. On admission he is unw ell and has a 

respiratory rate of 32 breaths per minute. His chest is wheezy on auscultation. 

Arterial blood gases show : 

pO2 9.5 kPa, pCO2 3.5 kPa, pH 7.48, HCO3 25, BE +2. 

 

What is the likely clinical scenario? 

A. Lactic acidosis secondary to metformin 

B. Anxiety disorder 

C. Asthma attack 

D. Chronic bronchitis 

E. Pneumothorax 

 

 

 

 

 

 

 

 

 

 

 

 

 

 

 

 

 

Answer: c) asthma attack.  

 

The patient has a respiratory alkalosis (pH > 7.45) which  is acute since the bicarbonate 

levels  are  normal  (22‐28).  There  is  hypoxia  and  the  patient  is  hyperventilating  in 

response and blowing off CO2. 

This would be consistent with an asthma attack. 

In an anxiety attack, hypoxia would not be present. 

www.MRCPass.com  Respiratory 

Dr. Khalid  Yusuf  Elzohry ‐ Sohag Teaching Hospital ‐ 2012 105

Respiratory Q102  

A 30 year old man  is admitted with a history of haemoptysis, cough and dyspnoea. He 

has been previously fit and has smoked 25 cigarettes a day for the last 10 years. A chest 

XR shows diffuse alveolar  infiltrates. He has a microcytic, hypochromic anaemia, urine 

dipstick  confirms haematuria and proteinuria. His  lung  function  tests  shows a normal 

spirometry and a TLCO of 125% predicted.  

 

What is the most likely diagnosis? 

A. Pulmonary emboli 

B. Chest trauma 

C. Goodpasture's syndrome 

D. Pulmonary tuberculosis 

E. Pneumonia 

 

 

 

 

 

 

 

 

 

 

 

 

 

 

Answer: c) Goodpasture's syndrome.  

 

Goodpasture's  syndrome  is  characterised  by  diffuse  alveolar  haemorrhage  and 

glomerulitis. Men are commonly affected with most cases occurring betw een the ages 

of  20‐30  years.  It  is more  likely  to  occur  in  smokers.  The  anti  glomerular  basement 

antibody  (Anti‐GBM)  is  present  in  up  to  90%  of  the  patients.  Renal  histology  usually 

shows a  focal segmental necrotizing glomerulitis with crescent  formation. The TLCO  is 

increased  during  active  bleeding  and  can  be  used  to  monitor  disease  activity.  An 

increase above 30% of baseline is suggestive of an intra‐alveolar bleed. 

www.MRCPass.com  Respiratory 

Dr. Khalid  Yusuf  Elzohry ‐ Sohag Teaching Hospital ‐ 2012 106

Respiratory Q103  

A 60 year old man presents with  sever breathlessness  following an upper  respiratory 

tract infection.  

 

Which of the following would support a diagnosis of acute respiratory distress syndrome 

(ARDS)? 

A. High protein content of pulmonary oedema fluid 

B. Normal chest Xray 

C. High pulmonary capillary wedge pressure 

D. Increased lung compliance 

E. High CO2 levels 

 

 

 

 

 

 

 

 

 

 

 

 

 

 

 

 

 

Answer: a) High protein content of pulmonary oedema fluid.  

 

Acute  respiratory  distress  syndrome  is  characterised  by  hypoxaemia,  reduced  lung 

compliance, pulmonary hypertension and pulmonary infiltrates on chest X ray. There is 

damage to the capillary endothelial cell  linings resulting  in oedema  leakage of proteins 

cells into interstitial alveolar spaces. 

A high pulmonary capillary wedge pressure suggests heart failure. High CO2 reflect type 

II respiratory failure and are non specific. 

www.MRCPass.com  Respiratory 

Dr. Khalid  Yusuf  Elzohry ‐ Sohag Teaching Hospital ‐ 2012 107

Respiratory Q104  

A 60 year old man had a transbronchial biopsy confirming squamous cell carcinoma of 

the lung.  

 

Which of the following is a contraindication tow ards surgical resection? 

A. Hypercalcaemia 

B. Superior vena caval obstruction 

C. Previous radiotherapy 

D. Metastasis to local lymph nodes 

E. Pleural effusion 

 

 

 

 

 

 

 

 

 

 

 

 

 

 

 

 

 

 

 

 

 

 

Answer: b) superior vena caval obstruction.  

 

Extensive nodal spread, distal metastases, stage  IIIB or more, and superior vena caval 

obstruction are contraindications tow ards surgery for lung cancer. 

www.MRCPass.com  Respiratory 

Dr. Khalid  Yusuf  Elzohry ‐ Sohag Teaching Hospital ‐ 2012 108

Respiratory Q105  

In which of the following have randomised controlled trials shown that long‐term oxygen 

therapy (LTOT) reduces mortality? 

A. Asthma 

B. Chronic obstructive lung disease 

C. Cryptogenic fibrosing alveolitis 

D. Cystic fibrosis 

E. Pulmonary sarcoidosis 

 

 

 

 

 

 

 

 

 

 

 

 

 

 

 

 

 

 

 

 

 

 

Answer: B) Chronic obstructive lung disease.  

 

There  is evidence  for a small  reduction  in mortality  in patients with COPD and resting 

hypoxia. Although indications for LTOT are largely based on mortality data, some studies 

have also  suggested  improvements  in other outcome measures,  including depression, 

cognitive function, quality of life, exercise capability, and frequency of hospitalisation. 

www 

Dr. K

Re 

A 50

She

 

Wha

 

 

 

Ans

 

Cau

Rais

w.MRCPass.c

Khalid  Yusuf

spiratory

0 year old w

 has had no

at is the like

A. Vagus

B. Horne

C. Hiatu

D. Phren

E. Hepat

wer: d) phr

uses of phre

pneumonia

pleurisy 

aortic aneu

substernal 

neoplasms 

thoracic su

herpes zost

vasculitis 

diabetes 

sed Left hem

com 

f  Elzohry ‐ So

y Q106 

woman has 

o history of 

ely diagnosi

s nerve palsy

er's syndrom

s hernia 

nic nerve pal

tomegaly 

renic nerve 

enic nerve p

urysm 

goiter 

rgery 

ter infection

midiaphragm

ohag Teachin

an incident

cardiothora

is? 

me 

lsy 

palsy. 

palsy are : 

ng Hospital ‐

tal finding o

acic surgery

‐ 2012 

of raised lef

y or trauma 

 

109 

ft hemidiap

to the ches

Res

hragm on t

st.  

spiratory

the CXR. 

www.MRCPass.com  Respiratory 

Dr. Khalid  Yusuf  Elzohry ‐ Sohag Teaching Hospital ‐ 2012 110

Respiratory Q107  

A 35 year old man has symptoms of wheezing and has been diagnosed as having  late 

onset asthma. How ever, his GP measured routine blood tests and found that he had a 

creatinine of 250 umol/l.  

 

What antibody is likely to be helpful in confirming the diagnosis? 

A. Antinuclear antibody 

B. Anti phospholipids antibody 

C. Anti Ro antibody 

D. Anti nuclear cytoplasmic antibody 

E. Anti gliadin antibody 

 

 

 

 

 

 

 

 

 

 

 

 

 

 

 

 

 

 

 

Answer: D) anti nuclear cytoplasmic antibody.  

 

The  clinical  diagnosis  is  likely  to  be  an  ANCA  positive  small  vessel  vasculitis  such  as 

polyarteritis  nodosa,  as  there  is  pulmonary  and  renal  involvement.  Churg  Strauss 

syndrome  should  also  be  considered  (only  a  small  proportion  of patients with Churg 

Strauss have a positive ANCA) 

www.MRCPass.com  Respiratory 

Dr. Khalid  Yusuf  Elzohry ‐ Sohag Teaching Hospital ‐ 2012 111

Respiratory Q108  

A 60 year old man has a  long history of smoking and COPD. His resting PO2  is 7.2 kPa 

and his continues to be breathless despite being on home nebulisers. He is assessed for 

long term oxygen therapy (LTOT).  

 

When is LTOT indicated? 

A. PO2 <7.2 kPa 

B. PO2 <7.8 kPa 

C. PO2 <8 kPa 

D. PO2 <8.5 kPa 

E. PO2 <9 kPa 

 

 

 

 

 

 

 

 

 

 

 

 

 

 

 

 

 

 

 

 

 

Answer: a) pO2 <7.2 kPa. 

 

When there is polycythaemia or pulmonary hypertension, Long Term Oxygen Therapy is 

indicated when pO2 < 8kPa. In uncomplicated COPD, it is indicated when pO2 < 7.2kPa 

www.MRCPass.com  Respiratory 

Dr. Khalid  Yusuf  Elzohry ‐ Sohag Teaching Hospital ‐ 2012 112

Respiratory Q109  

A 60 year old woman with asthma presents with a history of acute breathlessness and 

pleuritic chest pain. Her arterial blood gases show the following readings: 

pH 7.35, pO2 6.8 kPa, pCO2 4 kPa, bicarbonate 25 mmol/L 

 

Which test is most specific to acute pulmonary embolism? 

A. MRI of the chest 

B. CT pulmonary angiogram 

C. Chest x‐ray 

D. D‐Dimers 

E. V/Q scan 

 

 

 

 

 

 

 

 

 

 

 

 

 

 

 

 

 

 

 

 

 

Answer: b) CT pulmonary angiogram.  

 

Most of the tests are helpful but a CT pulmonary angiogram remains the gold standard 

diagnostic test for pulmonary embolism. 

www.MRCPass.com  Respiratory 

Dr. Khalid  Yusuf  Elzohry ‐ Sohag Teaching Hospital ‐ 2012 113

Respiratory Q110  

A 35  year old man presents with  swelling of his  lips and around  the  throat  following 

consumption of prawns. His 

investigations show : 

serum IgE 150 kU/L (0‐120) 

C3 level is 77 mg/dL (65‐190) 

C4 level is 45 mg/dL (15‐50) 

 

Which of the following diagnosis is likely? 

A. C1 esterase inhibitor deficiency 

B. Allergic reaction 

C. Systemic mastocytosis 

D. Moon face 

E. Cellulitis 

 

 

 

 

 

 

 

 

 

 

 

 

 

 

 

 

 

Answer: b) allergic reaction.  

 

Mildly  elevated  IgE  concentration  suggests  an  allergic  reaction  to  praw  ns.  In 

angioneurotic oedema due to C1 esterase inhibitor deficiency, a low C4 with normal C3 

level is seen (C2 is also low but not commonly measured). 

www.MRCPass.com  Respiratory 

Dr. Khalid  Yusuf  Elzohry ‐ Sohag Teaching Hospital ‐ 2012 114

Respiratory Q111  

A 40 year old man has with a 2 year history of  increasing shortness of breath. This  is 

worse with exertion e.g. climbing the stairs at home and is even slightly so at rest. He is 

a non‐smoker. 

On examination, his pulse  is 95 beats/min and his blood pressure  is 140/95 mmHg. He 

has finger clubbing and a crackling noise at the end of inspiration over the bases of the 

lungs. There was no ankle oedema Lung function tests show : 

FVC 2.5 l (predicted 3.2 ) 

FEV1 2.1 l (predicted 2.4) 

FEV1 % 76% (predicted 75%) 

Diffusing  Capacity:  Gas  Transfer  Factor  for  carbon  monoxide:  DLCO  17 

ml/min/mmHg (predicted) 25 

 

Which one of the following diagnosis is likely? 

A. Aspergilloma 

B. Guillain‐Barre syndrome 

C. Cryptogenic fibrosing alveolitis 

D. Asthma 

E. Congestive cardiac failure 

 

 

 

 

 

 

 

 

 

 

 

Answer: c) cryptogenic fibrosing alveolitis. 

 

A reduced FVC, with normal FEV1, FEV1% and PEF usually  indicates restriction of  lung 

volume . KCO (transfer factor) is also reduced in fibrotic lung disease, as in this case. The 

additional  findings  of  inspirational  crackles  and  clubbing  suggests  the  diagnosis  of 

cryptogenic fibrosing alveolitis. 

www.MRCPass.com  Respiratory 

Dr. Khalid  Yusuf  Elzohry ‐ Sohag Teaching Hospital ‐ 2012 115

Respiratory Q112  

A 60 year old man has severe COPD requiring home nebulisers and home oxygen. He is 

admitted with an infective exacerbation.  

 

Which of the following results would be expected on the arterial blood gases? 

A. PH 7.25 paCO2 7 paO2 7.5 HCO3 30 

B. PH 7.10 paCO2 7 paO2 4 HCO3 24 

C. PH 7.30 paCO2 4 paO2 8.5 HCO3 30 

D. PH 7.40 paCO2 4 paO2 4 HCO3 22 

E. PH 7.45 paCO2 3 paO2 12 HCO3 24 

 

 

 

 

 

 

 

 

 

 

 

 

 

 

 

 

 

 

 

 

 

Answer: a) pH 7.25 paCO2 7 paO2 7.5 HCO3 30.  

 

A long standing COPD patient would be expected to have a high bicarbonate. However 

this  patient  is  unwell  with  type  II  respiratory  failure  (high  CO2)  and  hence  has 

uncompensated respiratory acidosis. 

www 

Dr. K

Re 

A 60

and

Che

bac

 

Whi

tube

 

 

 

 

 

 

 

 

 

Ans

 

Slat

mac

Silic

w.MRCPass.c

Khalid  Yusuf

spiratory

0 year old m

 breathless

est XR show

illi.  

ich  of  the 

erculosis? 

A. Beryl

B. Cadm

C. Coal 

D. Silica 

E. House

wer: d) silic

te  workers,

crophage fu

cosis 

com 

f  Elzohry ‐ So

y Q113 

miner has b

ness. 

ws diffuse in

following 

lium 

mium 

e dust 

ca.  

  stonemas

unction, and

ohag Teachin

been in the

nterstitial sh

dusts  is 

ons  and m

d in particul

ng Hospital ‐

 occupation

hadowing. A

most  likely

miners  are  e

lar, predisp

‐ 2012 

n for 20 yea

A sputum s

y  to  have 

exposed  to

oses to TB i

 

116 

ars. He pres

ample is po

predispose

o  silica  dus

infection. 

Res

sents with a

ositive for a

ed  the  pat

t.  Silicosis 

spiratory

a cough 

acid fast 

tient  to 

impairs 

www.MRCPass.com  Respiratory 

Dr. Khalid  Yusuf  Elzohry ‐ Sohag Teaching Hospital ‐ 2012 117

Respiratory Q114  

A 34‐year‐old woman presented with a dry cough, thorax constriction, and generalised 

weakness.  During  the  preceding  5  months,  she  had  experienced  these  dry  cough 

episodes  tw  ice a week. A  lung  function  test showed a restriction of  the vital capacity 

(71% of the adjusted reference value), and the diffusion capacity was also reduced (66 

to 68%). 

Bronchoalveolar  lavage specimens were obtained,  the  lymphocytes were  increased up 

to  41%  (norm,  <10%),  and  neutrophils were  increased  up  to  6%  (norm,  <2%) with  a 

normal total cell count. 

 

What is the likely diagnosis? 

A. Asthma 

B. Pulmonary embolus 

C. Tuberculosis 

D. Extrinsic allergic alveolitis 

E. Alpha 1 antitrypsin deficiency 

 

 

 

 

 

 

 

 

 

 

 

 

Answer: d) extrinsic allergic alveolitis. 

 

Extrinsic Allergic Alveolitis is a type III or type IV response. There is no eosinophilia. IgG 

and  lymphocytes  are  involved  in  immune  response. Antigens of micropolyspora  faeni 

and  thermoactinomyces are 0.5‐5 microns  . These antigens which may be detected as 

serum precipitins. 

The acute form takes about 6 hours for sensitisation to the inhaled antigen. The chronic 

form may take weeks. 

www.MRCPass.com  Respiratory 

Dr. Khalid  Yusuf  Elzohry ‐ Sohag Teaching Hospital ‐ 2012 118

Respiratory Q115  

A 65 year old patient with COPD is on maximal treatment.  

 

Which one of the following is likely to prevent further disease progression? 

A. Steroids 

B. Beta agonist inhalers 

C. Stopping smoking 

D. Tiotropium 

E. Home oxygen 

 

 

 

 

 

 

 

 

 

 

 

 

 

 

 

 

 

 

 

 

 

 

 

Answer: C) stopping smoking.  

 

In  COPD,  discontinuation  of  smoking  is  the  only  features which  has  been  shown  to 

reduce disease progression. 

www.MRCPass.com  Respiratory 

Dr. Khalid  Yusuf  Elzohry ‐ Sohag Teaching Hospital ‐ 2012 119

Respiratory Q116  

A 60 year old male smoker has emphysema. 

His lung function tests show : 

FEV1 is 0.5 (20% predicted) 

FVC is 2.2 (61% predicted) 

FEV1:FVC ratio of 26% 

His  arterial blood gases  show  a PO2 of 7.5  and 7.2 on  two  separate occasions. He  is 

mildly breathless at rest, but severely breathless on exertion. 

 

What is the best measure of his respiratory function? 

A. TLCO 

B. FEV1 

C. FVC 

D. KCO 

E. TLC 

 

 

 

 

 

 

 

 

 

 

 

 

 

 

Answer: b) FEV1. 

 

The  breathlessness  worsens  considerably  without  much  change  in  oxygen  tension, 

suggesting that the cause of his dyspnoea  is hyperinflation of his chest which worsens 

on exertion. 

Severity  of  emphysema  is  defined  by  the British  Thoracic  Society  (BTS)  in  relation  to 

FEV1, not FEV1:FVC ratio. Mild is 60‐80% predicted; moderate 40‐60% and severe <40%. 

www.MRCPass.com  Respiratory 

Dr. Khalid  Yusuf  Elzohry ‐ Sohag Teaching Hospital ‐ 2012 120

Respiratory Q117  

A  40  year  old  lady  was  admitted  to  hospital  with  fevers  and  cough  productive  of 

sputum.  

Chest X‐ray shows diffuse patchy consolidation around the  left  lung. She has had a  flu 

like  illnes 4 weeks ago, and has a past medical history of asthma. She also smokes 10 

cigarettes a day.  

 

Which organism is likely to be responsible? 

A. Mycoplasma 

B. Pseudomonas 

C. Klebsiella 

D. Staphylococcus 

E. Tuberculosis 

 

 

 

 

 

 

 

 

 

 

 

 

 

 

 

 

 

 

Answer: d) staphylococcus.  

 

Following  a  viral  infection,  patients  are  predisposed  to  staphylococcal  infection.  The 

chest XR changes suggest staphlococcus rather than streptococcus (which would cause 

lobar consolidation). 

www 

Dr. K

Re 

A 50

apy

His 

mm

(0.8

Che

 

Wha

 

 

 

 

 

 

 

Ans

 

Sarc

hila

effu

Bila

w.MRCPass.c

Khalid  Yusuf

spiratory

0 year old m

rexial on ad

blood  sho

mol/l, potass

8‐1.4)mmol/

est X ray sho

at is the like

A. Extrin

B. Tuber

C. Sarco

D. Allerg

E. Leiom

wer: c) sarc

coidosis can

r  lymphade

usions and n

teral hilar ly

com 

f  Elzohry ‐ So

y Q118 

man presen

dmission. 

w  ESR  60 

sium 4.1 mm

/l. 

ows bilatera

ely diagnosi

nsic allergic a

rculosis 

idosis 

gic bronchop

myoma 

coidosis.  

n cause ma

enopathy, 

nodules. 

ymphadeno

ohag Teachin

ts with a 3 

mm/hr,  ur

mol/l, corre

al hilar lymp

is? 

alveolitis 

pulmonary as

ny changes

diffuse  par

opathy in sa

ng Hospital ‐

month hist

rea  7  μmo

ected calciu

phadenopat

spergillosis

s on the CXR

renchymal 

 arcoidosis 

‐ 2012 

ory of coug

l/l,  creatin

m 2.75 (2.2

thy and egg

R. Among th

changes,  e

121 

gh and brea

ine  100  μm

2‐2.7) mmo

gshell calcifi

hese are un

eggshell  ca

Res

thlessness. 

mol/l,  sodiu

l/l, phospha

cation. 

nilateral or b

alcification, 

spiratory

He was 

um  137 

ate 0.82 

bilateral 

pleural 

www.MRCPass.com  Respiratory 

Dr. Khalid  Yusuf  Elzohry ‐ Sohag Teaching Hospital ‐ 2012 122

Respiratory Q119  

A 40 year old woman presents with breathlessness, cough and  fever. On examination, 

she has basal crackles in the lung fields. Circulating precipitans to Micropolyspora faeni 

are positive.  

 

Which of the following is the most likely diagnosis? 

A. Pigeon fanciers' lung 

B. Allergic Bronchopulmonary Aspergillosis 

C. Farmers' lung 

D. PCP infection 

E. Pulmonary fibrosis 

 

 

 

 

 

 

 

 

 

 

 

 

 

 

Answer: c) Farmers' lung.  

 

Farmer's  lung  is  the  most  common  type  of  hypersensitivity  pneumonitis. 

Hypersensitivity  pneumonitis,  also  known  as  extrinsic  allergic  alveolitis,  is  associated 

with intense or repeated exposure to inhaled biologic dusts. The classic presentation of 

farmer's  lung  results  from  inhalational  exposure  to  thermophilic Actinomyces  species 

and occasionally from exposure to various Aspergillus species. 

Acute  farmer's  lung  develops  after  large  exposure  to  moldy  hay  or  contaminated 

compost.  Symptoms  often  spontaneously  resolve within  12  hours  to  days  if  antigen 

exposure  is  eliminated  or  avoided.  It  manifests  as  new  onset  of  fever,  chills, 

nonproductive cough, chest tightness, dyspnea, headache, and malaise. 

www.MRCPass.com  Respiratory 

Dr. Khalid  Yusuf  Elzohry ‐ Sohag Teaching Hospital ‐ 2012 123

Respiratory Q120  

A  50  year  old meat  factory worker  has  been  unw  ell with  fever,  cough,  sweats  and 

lethargy. On examination, he had a purpuric rash and hepatosplenomegaly.  

 

What is the likely diagnosis? 

A. Pulmonary embolism 

B. Respiratory syncytial viral pneumonia 

C. Streptococcal pneumonia 

D. Q fever pneumonia 

E. Aspergilloma 

 

 

 

 

 

 

 

 

 

 

 

 

 

 

 

 

 

 

 

Answer: d) Q fever pneumonia. 

 

Q  fever  is due  to Coxiella burnetii and  is acquired  via animal  contact.  It  can occur  in 

outbreaks  in  farming  communities  and  in  abbatoirs.  Treatment  is  with  prolonged 

courses of tetracyclines. Rarely infection can be persistent leading to chronic symptoms 

including  fatigue, malaise  and  sweats. Hepatitis,  hepatosplenomegaly, maculopapular 

rash and endocarditis are associated. 

www 

Dr. K

Re 

A 5

had

felt 

che

prot

 

The

 

 

 

 

 

Ans

 

The

Exa

>30

Emp

w.MRCPass.c

Khalid  Yusuf

spiratory

0 year old 

 suffered a

better  init

st  radiogra

tein of 42 g

 most likely

A. Collap

B. Empy

C. Serou

D. Fibro

E. Tuber

wer: b) Emp

e  likely  cau

mination o

0g would be

pyema (air f

com 

f  Elzohry ‐ So

y Q121 

woman,  is 

 chest infec

tially, but  is

ph  shows 

g and pH of 

y diagnosis i

pse of the le

yema 

us pleural eff

tic lung disea

rculosis 

pyema.  

use  is  a  p

f  a pleural 

 consistent 

fluid level) 

ohag Teachin

admitted w

ction for wh

s now  getti

a  left  sided

7.0.  

is: 

ft lung 

fusion 

ase 

pneumonia

aspirate  (m

with an exu

ng Hospital ‐

with malais

hich she wa

ing worse,

d  pleural  e

which  has

microscopy

udate and a

‐ 2012 

se and  feve

as given a c

complainin

ffusion.  An

s  not  reso

for organis

a pH of < 7.2

 

124 

er. Four we

ourse of or

ng of  interm

n  aspirate  s

olved,  lead

sms,  cultur

2 is suggest

Res

eks previou

ral amoxyci

mittent  feve

shows  fluid

ing  to  em

re). A prote

tive of infec

spiratory

usly  she 

llin. She 

ers. Her 

 with  a 

mpyema. 

ein  level 

ction.